Test 3 Saunders & Davis NCLEX questions

Ace your homework & exams now with Quizwiz!

A child must wear a brace for correction of scoliosis. The nurse develops a plan of care knowing the child is at risk for which problem? 1. Inability to ambulate 2. Breaks in skin integrity 3. Decreased oxygenation 4. Delayed growth and development

2 Braces for treatment of scoliosis usually are worn 16 to 23 hours a day. The skin should be kept clean and dry and inspected for signs of redness or breakdown. Therefore breaks in skin integrity are the client problem that should be included in this child's plan of care. The brace assists with posture, so mobility is not an issue. The brace does not compromise the respiratory status, so oxygenation is not decreased. The child will not have a risk for delayed growth and development because normal developmental milestones can be met while wearing a brace.

An adolescent with diabetes receives 30 units of Humulin N insulin at 7:00 am. In accordance with the peak insulin action time, the nurse would monitor for a hypoglycemic episode at what time? 1. At bedtime 2. At midmorning 3. Before supper 4. After breakfast

3 Humulin N insulin is an intermediate-acting insulin that peaks in approximately 6 to 12 hours. It would peak before supper if given at 7:00 am. Short-acting insulin would peak after breakfast or midmorning. Long-acting insulins would peak at bedtime.

The family of a young child has been told the child has diabetes insidious (DI). What information should the nurse emphasize to the family? 1. One caregiver needs to learn to give the injections of vasopressin. 2. Children should wear Medic Alert tags if they are over 5 years old. 3. Diabetes insipidus is different from diabetes mellitus. 4. Over time, the child may grow out of the need for medication.

3. Explaining that DI is different from DM is crucial to the parents' understanding of the management of the disease. DI is a rare condition that affects the posterior pituitary gland, whereas DM is a more common condition that affects the pancreas.

A 7-year-old is tested for DI. 24 hours after his fluid restriction has begun, the nurse notes that his urine continues to be clear and pale, with a low specific gravity. Which is the most likely reason for this? 1. 24 hours is too early to evaluate effects of fluid restriction. 2. The urine should be concentrated, and it is unlikely the child has DI. 3. The child may have been sneaking fluids and needs closer observation. 4. In DI, fluid restriction does not cause urine concentration.

4. Children with DI cannot concentrate urine.

A child with nephrotic syndrome has been prescribed prednisone. The nurse should monitor the child for which of the following medication side effects? 1. Gastric distress 2. Bradycardia 3. Hypoglycemia 4. Weight loss

1. Gastric distress is a common side effect of prednisone. Bradycardia & weight loss are not documented side effects of prednisone. Hyperglycemia is seen in patients taking high doses of prednisone.

A child with type 1 diabetes mellitus has been diagnosed with ketoacidosis. Which of the following laboratory findings consistent with the diagnosis? 1. Hemoglobin AIC 5.5% 2. Fasting blood glucose 124 mg/dl 3. Serum PH: 7.24 4. Potassium level: 3.9 mEq/L

3. This is indicative of ketoacidosis. The others are normal findings.

A child, who is frightened, is hyperventilating. Which of the following blood gas values would the nurse expect to see? SATA 1. Depressed Pco2 2. Depressed Po2 3. Elevated PH 4. Elevated HCO3 5. Base excess of 0

1, 3 The nurse would expect to see a low Pco2- and an elevated PH.

A child is admitted to the hospital with diarrhea, vomiting and dehydration. One week earlier, the child weighed 5.6kg. On admission to the hospital, the child weighs 4.9 kg. What percentage weight loss has the child experienced? Please calculate to the tenths place. ___________________%

12.5% 5.6 kg previous weight -4.9 kg new weight ____ 0.7 difference (0.7 + 5.6) x 100= 0.125 x 100 = 12.5%

A child is hospitalized because of persistent vomiting. The nurse should monitor the child closely for which problem? 1. Diarrhea 2. Metabolic acidosis 3. Metabolic alkalosis 4. Hyperactive bowel sounds

3 Vomiting causes the loss of hydrochloric acid and subsequent metabolic alkalosis. Metabolic acidosis would occur in a child experiencing diarrhea because of the loss of bicarbonate. Diarrhea might or might not accompany vomiting. Hyperactive bowel sounds are not associated with vomiting.

The nurse understands that which information collected during the assessment of a child recently diagnosed with glomerulonephritis is most often associated with the diagnosis? 1. Child fell off a bike onto the handlebars 2. Nausea and vomiting for the last 24 hours 3. Urticaria and itching for 1 week before diagnosis 4. Streptococcal throat infection 2 weeks before diagnosis

4 Glomerulonephritis refers to a group of kidney disorders characterized by inflammatory injury in the glomerulus. Group A b-hemolytic streptococcal infection is a cause of glomerulonephritis. Often, a child becomes ill with streptococcal infection of the upper respiratory tract and then develops symptoms of acute poststreptococcal glomerulonephritis after an interval of 1 to 2 weeks. The assessment data in options 1, 2, and 3 are unrelated to a diagnosis of glomerulonephritis.

A nurse provides instructions to the parents of an infant with hip dysplasia regarding care of the Pavlik harness. Which statement by one of the parents indicates an understanding of the use of the harness? 1. "I can remove the harness to bathe my infant." 2. "I need to remove the harness to feed my infant." 3. "I need to remove the harness to change the diaper." 4. "My infant needs to remain in the harness at all times."

1 The harness should be worn 23 hours a day and should be removed only to check the skin and for bathing. The hips and buttocks should be supported carefully when the infant is out of the harness. The harness does not need to be removed for diaper changes or feedings. Option 4 is incorrect.

The parents of a child, whose weight is 64 lb, are advised to make sure that the child consumes the minimum fluid needed to maintain a normal hydration status. Please calculate the baby's needs to the nearest whole number. _____________________ ml/day

1,682 ml per day 2.2 lb/1 kg= 64 lb/x kg 2.2x= 64 x= 29.09 kg 10 kg x 100 ml/kg = 1,000 ml 10 kg x 50 ml/kg = 500 ml 9.09 kg x 20 ml/kg = 1,81.8 ml Total daily minimum= 1,681.8 or 1682 ml per day

A teenage child has been diagnosed with type 2 diabetes The nurse determines that the child will likely be administered which of the following medications? 1. Metformin (Glucophage) 2. Aspart (Novolog) 3. Detemir (Levemir) 4. Glargine (Lantus)

1. Metformin (Glucophage) is usually the first line drug for patients with type 2 diabetes. Aspart is a short acting injectable used for type 1 diabetes Determir is an injectable intermediate acting insulin used for type 1 diabetes Glargine is an injectable intermediate acting insulin used for type 1 diabetes

The nurse assesses the following blood gas results on an infant in the emergency department. Which of the following conclusions is consistent with the data? Po2- 90 mm Hg Pco2- 34 mm Hg HCO3- 16 mm Hg Base excess -4 PH 7.28 1. Metabollic acidosis 2. Metabolic alkalosis 3. Respiratory acidosis 4. Respiratory alkalosis

1. The child is in metabolic acidosis

After performing an assessment of an infant with bladder exstrophy, a nurse prepares a plan of care. The nurse identifies which problem as the priority for the infant? 1. Urinary incontinence 2. Impaired tissue integrity 3. Inability to suck and swallow 4. Lack of knowledge about the disease (parents)

2 In bladder exstrophy, the bladder is exposed and external to the body. The highest priority is impaired tissue integrity related to the exposed bladder mucosa. Although the infant needs to be monitored for elimination patterns and kidney function, urinary incontinence is not a concern for this condition as the infant is not yet toilet trained. Inability to suck and swallow is unrelated to the disorder. Lack of knowledge about the diagnosis and treatment of the condition will need to be addressed but again is not the priority.

A 7-month-old child has been diagnosed with cerebral palsy. Which of the following signs/symptoms would the nurse assess as consistent with the diagnosis? 1. Positive grasp reflex 2. Pigeon chest 3. Harlequin sign 4. Circumoral cyanosis

1 Positive grasp reflex would be consistent with the diagnosis. The other options are unrelated to a diagnosis of CP.

A nurse is developing a plan of care for a 10-year-old child diagnosed with acute glomerulonephritis. What is the priority nursing intervention? 1. Promoting bed rest 2. Restricting oral fluids 3. Allowing the child to play 4. Encouraging visits from friends

1 Bed rest is required during the acute phase, and activity is gradually increased as the condition improves. Fluids should not be forced or restricted. Providing for quiet play according to the developmental stage of the child is important. Visitors should be limited to allow for adequate rest

The nurse is reviewing the laboratory test results for an infant suspected of having hypertrophic pyloric stenosis. The nurse should expect to note which value as the most likely laboratory finding in this infant? 1. Blood pH of 7.50 2. Blood pH of 7.30 3. Blood bicarbonate of 22 mEq/L 4. Blood bicarbonate of 19 mEq/L

1 Laboratory findings in an infant with hypertrophic pyloric stenosis include metabolic alkalosis due to vomiting. These include increased blood pH and bicarbonate level, decreased serum potassium and sodium levels, and a decreased chloride level. The normal pH is 7.35 to 7.45. The normal bicarbonate is 22 to 27 mm Hg.

A child has been diagnosed with Hirschsprung's disease. Which of the following findings would the nurse expect the parents to report in the child's history? SATA 1. Ribbon-like stools 2. Chronic constipation 3. Black and tarry stools 4. Distended abdomen 5. Delayed meconium passage

1,2,4 & 5 These are all indicative of Hirschsprung's disease.

The nurse is caring for a newborn infant after surgical intervention for imperforate anus. The nurse should place the infant in which position in the postoperative period? 1. Supine with no head elevation 2. Side-lying with the legs flexed 3. Side-lying with the legs extended 4. Supine with the head elevated 30 degrees

2 After surgical intervention for imperforate anus, a side-lying position with the legs flexed or a prone position to keep the hips elevated can reduce edema and pressure on the surgical site. Options 1, 3, and 4 are incorrect positions.

The mother of an infant diagnosed with Hirschsprung's disease asks the nurse about the disorder. What should the nurse tell the mother about the disease? 1. It is complete small intestinal obstruction. 2. It is congenital aganglionosis or megacolon. 3. It is severe inflammation of the gastrointestinal tract. 4. It is condition that causes the pyloric valve to remain open.

2 Hirschsprung's disease is also known as congenital aganglionosis or megacolon. It is the result of an absence of ganglion cells in the rectum and to varying degrees upward in the colon. The remaining options are incorrect descriptions.

A mother brings her 3-week-old infant to a clinic for a phenylketonuria rescreening blood test. The test indicates a serum phenylalanine level of 1 mg/dL. The nurse reviews this result and makes which interpretation? 1. It is positive. 2. It is negative. 3. It is inconclusive. 4. It requires rescreening at age 6 weeks.

2 Phenylketonuria is a genetic (autosomal recessive) disorder that results in central nervous system damage from toxic levels of phenylalanine (an essential amino acid) in the blood. It is characterized by blood phenylalanine levels greater than 20 mg/dL (normal level is 1.2 to 3.4 mg/dL in newborns and 0.8 to 1.8 mg/dL thereafter). A result of 1 mg/dL is a negative test result.

A 6-year-old child with diabetes mellitus and the child's mother come to the health care clinic for a routine examination. The nurse evaluates the data collected during this visit to determine if the child has been euglycemic since the last visit. Which information is the most significant indicator of euglycemia? 1. Daily glucose monitor log 2. Glycosylated hemoglobin (hemoglobin A1c) 3. Dietary history for the previous week 4. Fasting blood glucose performed on the day of the clinic visit

2 The glycosylated hemoglobin assay measures the glucose molecules that attach to the hemoglobin A molecules and remain there for the life of the red blood cell, approximately 120 days. This is not reversible and cannot be altered by human intervention. Daily glucose logs are useful if they are kept regularly and accurately. However, they reflect only the blood glucose at the time the test was done. A fasting blood glucose test performed on the day of the clinic visit is time-limited in its scope, as is the dietary history.

The nurse is providing instructions to the parents of a child with scoliosis regarding the use of a brace. Which statement by the parents indicates a need for further instruction? 1. "I will encourage my child to perform prescribed exercises." 2. "I will have my child wear soft fabric clothing under the brace." 3. "I should apply lotion under the brace to prevent skin breakdown." 4. "I should avoid the use of powder because it will cake under the brace."

3 A brace may be prescribed to treat scoliosis. Braces are not curative, but may slow the progression of the curvature to allow skeletal growth and maturity. The use of lotions or powders under a brace should be avoided because they can become sticky and cake under the brace, causing irritation. Options 1, 2, and 4 are appropriate interventions in the care of a child with a brace.

The nurse is providing instructions to the parents of a child with a hernia regarding measures that will promote reducing the hernia. The nurse determines that the parents understand care for their child if they make which statement? 1. "We will encourage our child to cough every few hours on a daily basis." 2. "We will make sure that our child participates in physical activity every day." 3. "We will provide comfort measures to reduce any crying periods by our child." 4. "We will be sure to give our child a Fleet enema every day to prevent constipation."

3 A warm bath, avoidance of upright positioning, and other comfort measures to reduce crying are all simple measures to reduce a hernia. Coughing and crying increase the strain on the hernia. Likewise, physical activity and enemas of any type would increase the strain on the hernia.

The nurse reviews the record of a newborn infant and notes that a diagnosis of esophageal atresia with tracheoesophageal fistula is suspected. The nurse expects to note which most likely sign of this condition documented in the record? 1. Incessant crying 2. Coughing at nighttime 3. Choking with feedings 4. Severe projectile vomiting

3 In esophageal atresia and tracheoesophageal fistula, the esophagus terminates before it reaches the stomach, ending in a blind pouch, and a fistula is present that forms an unnatural connection with the trachea. Any child who exhibits the "3 C's"-coughing and choking with feedings and unexplained cyanosis-should be suspected to have tracheoesophageal fistula. Options 1, 2, and 4 are not specifically associated with tracheoesophageal fistula.

The nurse is reviewing the health care provider's prescriptions for a child hospitalized with nephrotic syndrome. Which dietary prescription should the nurse expect to be prescribed for the child? 1. A high-protein, high-salt diet 2. A full liquid diet for 1 month 3. A low-fat, high carbohydrate diet 4. A normal protein, mild sodium diet

4 For the child with nephrotic syndrome, a diet that is normal in protein, with a mild sodium restriction (to reduce fluid retention), is normally prescribed. Options 1, 2, and 3 are incorrect diets for this child.

The nurse is assisting a health care provider (HCP) during the examination of an infant with developmental hip dysplasia. The health care provider performs the Ortolani maneuver. The nurse determines that the infant exhibits a positive response to this maneuver if which finding is noted? 1. A shrill cry from the infant 2. Asymmetry of the affected hip 3. Reduced range of motion in the right and left hip 4. A palpable click during abduction of the affected hip

4 In the Ortolani maneuver, the examiner abducts both hips. A positive finding is a palpable click on the affected side during abduction. Crying is expected. Asymmetry and reduced range of motion of the affected hip are not positive signs of this maneuver.

The mother of a child with hepatitis A tells the home care nurse that she is concerned because the child's jaundice seems worse. Which response should the nurse make to the mother? 1. "You need to change the child's diet." 2. "The child probably is infectious again." 3. "You need to call the health care provider." 4. "In many situations, the jaundice worsens before it resolves."

4 The parents of the child should be told that jaundice may appear to worsen before it resolves. Options 1, 2, and 3 are incorrect and inappropriate responses.

The nurse is providing a yearly summer educational session to parents in a local community. The topic of the session is prevention and treatment measures for poison ivy. The nurse instructs the parents that if the child comes into contact with poison ivy to take which action? 1. Immediately report to the emergency department. 2. Avoid becoming concerned if a rash is not noted on the skin. 3. Apply calamine lotion immediately to the exposed skin areas. 4. Shower the child immediately, lathering and rinsing the exposed skin several times.

4 When a person comes into contact with a poison ivy plant, the sap from the plant forms an invisible film on the human skin. The client should be instructed to shower immediately; the skin should be lathered several times and rinsed each time in running water. Application of calamine lotion is a treatment that is used if dermatitis develops. It is not necessary for the client to be seen in the emergency department at the time of initial contact with poison ivy.

The nurse is initiating nasogastric tube feedings in a child. When initiating this procedure the nurse should perform which action? 1. Microwave the formula. 2. Place the child in a prone position. 3. Encourage the child to point the head downward. 4. Position the child so that the head is slightly hyperflexed.

4 When initiating nasogastric tube feedings a child, he or she should be positioned so that the head is slightly hyperflexed or in a sniffing position with the nose pointed toward the ceiling. The formula should be warmed to room temperature, and a microwave should not be used.

A child is diagnosed with acute glomerular nephritis. Which of the following changes would the nurse expect to see in the child's lab reports? 1. Urine white blood cell count; elevated 2. Urine specific gravity; decreased 3. Urine creatinine clearance; decreased 4. Urine RBC count; elevated

4. The number of red blood cells in the urine increases dramatically. The nurse would expect to see WBCs in the urine if the child had a UTI. Because of the hematuria and proteinuria, the nurse would expect to see an increase in the child's urinary specific gravity. Because the child's kidney function is compromised, the nurse would expect to see reduced creatinine clearance in the urine, but a concurrent rise in the serum creatinine.

A one-month-old baby, 8 lb 4 oz, is in the hospital with a diagnosis of pyloric stenosis. The nurse is carefully assessing the child's intake and output. Please calculate the minimum urinary output the baby should excrete per hour. Please calculate to the nearest tenth. _____________ ml/hr

7.5 ml/hr The minimum output ml per hour of an infant is equal to 2 ml times the baby's weight in kg. A baby that weighs 8 lb. 4 oz. weights 3.75 kg. Two times 3.75 equals a minimum hourly output of 7.5 ml.

A young boy who has been diagnosed with growth hormone deficiency is to receive synthetic growth hormone. When providing medication teaching to the boy and his parents, which of the following information should the nurse include? 1. Educate the boy and his parents regarding the rationale for the administration of the subcutaneous injections. 2. Advise the boy to immediately report signs and symptoms of gynecormastia. 3. Advise the boy that he will reach his desired height if he takes the medication as ordered. 4. Educate the boy that to maintain his height, he will have to take the medications for the rest of his life.

1..This statement is correct. The child will receive GH subcutaneous injections at bedtime six to seven times per week. The other answers are incorrect: Gynecomastia is not seen with GH injections, Even with injections the boy may not reach his desired height & the mediation is taken until either the child reaches his desired height or the growth plates fuse.

The nurse is reviewing the record of a child diagnosed with nephrotic syndrome. The nurse should expect to note which finding documented in the child's record? 1. Polyuria 2. Weight gain 3. Hypotension 4. Grossly bloody urine

2 Massive edema resulting in dramatic weight gain is a characteristic finding in nephrotic syndrome. Urine is dark, foamy, and frothy, but only microscopic hematuria is present; frank bleeding does not occur. Urine output is decreased, and hypertension is likely to be present.

A baby was just born with a gastroschisis. Which of the following actions by the nurse is priority? 1. Inform the parents regarding the etiology of the defect. 2. Cover the defect with a moist, sterile dressing. 3. Administer IV antibiotics, as ordered. 4. Educate the parents regarding the surgical repair.

2. It is a priority for the nurse to cover the defect with a moist, sterile dressing. The other needs are important but not a priority.

A child is admitted to the pediatric unit with nephrotic syndrome. Which of the following lab results would the nurse expect to see? 1. Thrombocytopenia 2. Hypoalbuminemia 3. Neutropenia 4. Hypermagnesemia

2. The child's serum albumin levels would be markedly decreased. The nurse would expect the other options to be within normal limits.

A 10-year-old child is diagnosed with enterobiasis (pinworm). Which of the following signs/symptoms would the nurse expect to see? 1. Recurrent vomiting 2. Enuresis 3. Bloody diarrhea 4. Pain

2. The nurse would expect the child to be wetting the bed. The other options would not be expected.

A nurse is educating a young boy about the assessments required to make a diagnosis of growth hormone deficiency. Which of the following information should the nurse include in his or her teaching? 1. A biopsy of the child's testes will be conducted. 2. An x-ray of the child's wrists will be be performed. 3. The child will have an MRI of his hypothalamus. 4. The child will receive IV dye for the adrenal fluoroscopy.

2. an x-ray of the child's wrists will be performed.

A baby who weighs 4.8kg is in the hospital. The child's hydration status is within normal limits. The nurse is calculating the minimum volume of fluid the child needs per hour to maintain normal hydration status. Please calculate the baby's needs to the nearest whole number. ____________________ ml/hr

20 ml/hr 4.8 kg x 100 ml/kg=480 ml, daily minium fluid needs 480 ml/24 hr= x ml/hr x= 20 ml/hr

An adolescent is examined in the hospital emergency department after taking an overdose of acetylsalicylic acid (aspirin). The adolescent has rapid breathing, nausea and vomiting, and lethargy. The health care provider prescribes arterial blood specimens for blood gas analysis to be drawn. Aspirin toxicity is suspected when the blood gas results are reported as which value? 1. pH 7.50, Pco2 60, HCO3 30 2. pH 7.44, Pco2 30, HCO3 21 3. pH 7.29, Pco2 29, HCO3 19 4. pH 7.33, Pco2 52, HCO3 28

3 The client who has aspirin toxicity will manifest metabolic acidosis with respiratory compensation as seen when the pH is lower 7.35 mm Hg, and the HCO3 is less than 22 mEq/L. In the correct option, the pH is acidotic and the HCO3 is decreased, indicating metabolic acidosis. The Pco2 is alkalotic, indicating partial compensation.

The nurse has just administered ibuprofen (Motrin) to a child with a temperature of 38.8° C (102° F). The nurse should also take which action? 1. Withhold oral fluids for 8 hours. 2. Sponge the child with cold water. 3. Plan to administer salicylate (aspirin) in 4 hours. 4. Remove excess clothing and blankets from the child.

4 After administering ibuprofen, excess clothing and blankets should be removed. The child can be sponged with tepid water, but not cold water because the cold water can cause shivering, which increases metabolic requirements above those already caused by the fever. Aspirin is not administered to a child with fever because of the risk of Reye's syndrome. Fluids should be encouraged to prevent dehydration, so oral fluids should not be withheld.

The parents of a child with a cleft palate are concerned and ask the nurse when the palate will be repaired. The nurse should plan to base the response on which information about cleft palate repair? 1. Cannot be repaired 2. Repair usually is performed by age 8 weeks 3. Repair usually is performed by 2 months of age 4. Repair usually is performed between 6 months and 2 years

4 Cleft palate repair is individualized and is based on the degree of deformity and size of the child. Cleft palate repair usually is performed between 6 months and 2 years of age, depending on the preference of the health care provider. Early closure facilitates speech development. Options 1, 2, and 3 are incorrect.

A 3-year-old child is seen in the health care clinic, and a diagnosis of encopresis is made. The nurse reviews the assessment findings and expects to note documentation of which sign of this disorder? 1. Diarrhea 2. Malaise and anorexia 3. Nausea and vomiting 4. Evidence of soiled clothing

4 Encopresis is defined as fecal incontinence and is a major concern if the child is constipated. Signs include evidence of soiled clothing, scratching or rubbing the anal area because of irritation, fecal odor without apparent awareness by the child, and social withdrawal.

A nurse has reinforced teaching for a school-age child who was given a brace to wear for the treatment of scoliosis. The nurse determines that the child needs further teaching if the child makes which statement? 1. "This brace will correct my curve." 2. "I will wear my brace under my clothes." 3. "I may not need surgery if I wear my brace." 4. "I will do back exercises at least five times a week."

1 Bracing can halt the progression of most curvatures, but it is not curative for scoliosis. The statements in options 2, 3, and 4 represent correct understanding on the part of the child.

A nurse is providing education to 4 sets of parents whose children have been diagnosed with type 1 diabetes. The nurse should provide follow-up education to the parents who state that they will perform which of the following actions? 1. Parents of a 2-year old: "We will have our daughter prick her finger for each glucose testing." 2. Parents of a 5-year old: "We will giver our daughter a code word that she will say when she feels a hypoglycemic episode developing." 3. Parents of a 9-year old: "We will monitor our daughter as she draws up and administers her insulin injections." 4. Parents of a 17-year old: "WE will allow our daughter to take responsibility for all her own diabetic care."

1. Two year olds are too young to prick their own fingers for glucose testing. The other options are appropriate

The nurse is preparing to care for a newborn infant who will be returning from surgery with a colostomy that was created for imperforate anus. When the infant arrives, the nurse assesses the stoma and notes that it is red and edematous. Which is the most appropriate nursing intervention? 1. Elevate the buttocks. 2. Document the findings. 3. Apply ice immediately. 4. Call the health care provider.

2 A fresh colostomy stoma will be red and edematous, but this will decrease with time. The colostomy site will then be pink without evidence of abnormal drainage, swelling, or skin breakdown. The nurse would document these findings because they represent a normal expectation. The remaining options are inappropriate interventions.

The nurse reviews the record of a child who is suspected to have glomerulonephritis and expects to note which finding that is associated with this diagnosis? 1. Hypotension 2. Brown-colored urine 3. Low urinary specific gravity 4. Low blood urea nitrogen level

2 Glomerulonephritis refers to a group of kidney disorders characterized by inflammatory injury in the glomerulus. Gross hematuria, resulting in dark, smoky, cola-colored or brown-colored urine, is a classic symptom of glomerulonephritis. Hypertension is also common. Blood urea nitrogen levels may be elevated. A moderately elevated to high urinary specific gravity is associated with glomerulonephritis.

The nurse is caring for an infant with a diagnosis of bladder exstrophy. To protect the exposed bladder tissue, the nurse should plan which intervention? 1. Cover the bladder with petroleum jelly gauze. 2. Cover the bladder with a nonadhering plastic wrap. 3. Apply sterile distilled water dressings over the bladder mucosa. 4. Keep the bladder tissue dry by covering it with dry sterile gauze.

2 In bladder exstrophy, the bladder is exposed and external to the body. In this disorder, one must take care to protect the exposed bladder tissue from drying, while allowing the drainage of urine. This is accomplished best by covering the bladder with a nonadhering plastic wrap. The use of petroleum jelly gauze should be avoided because this type of dressing can dry out, adhere to the mucosa, and damage the delicate tissue when removed. Dry sterile dressings and dressings soaked in solutions (that can dry out) also damage the mucosa when removed.

A nurse is caring for an infant with gastroenteritis who is being treated for dehydration. The nurse reviews the health record and notes that the health care provider has documented that the infant is mildly dehydrated. Which assessment finding should the nurse expect to note in mild dehydration? 1. Anuria 2. Pale skin color 3. Sunken fontanels 4. Dry mucous membranes

2 In mild dehydration the skin color is pale. Options 1 and 3 are assessment characteristics of severe dehydration. Option 4 is an assessment characteristic of moderate dehydration.

The nurse is developing a plan of care for an infant being admitted with hypertrophic pyloric stenosis who is scheduled for pyloromyotomy. In the preoperative period, which position should the nurse suggest to document in the plan of care? 1. In an infant seat placed in the crib 2. Prone with the head of the bed elevated 3. Supine with the head at a 90-degree angle 4. Supine with the head of the bed at a 30-degree angle

2 In the preoperative period, the infant is positioned prone with the head of the bed elevated to reduce the risk of aspiration. Options 1, 3, and 4 are inappropriate positions for preventing this risk.

An infant is seen in the health care provider's office for complaints of projectile vomiting after feeding. Findings indicate that the child is fussy and is gaining weight but seems to never get enough to eat. Pyloric stenosis is suspected. Which prescription would the nurse anticipate having the highest priority in the care of this child? 1. Administer predigested formula. 2. Prepare the family for surgery for the child. 3. Administer omeprazole (Prilosec) before feeding. 4. Instruct the parents to keep a log of feedings and any reflux present.

2 Infants with projectile vomiting after feeding that are fussy should be suspected of pyloric stenosis. The treatment for this diagnosis is surgery. The other options are treatment measures that may be prescribed for gastroesophageal reflux.

The nurse is reviewing the laboratory results for an infant with suspected hypertrophic pyloric stenosis. What should the nurse expect to note as the most likely finding in this infant? 1. Metabolic acidosis 2. Metabolic alkalosis 3. Respiratory acidosis 4. Respiratory alkalosis

2 Laboratory findings in an infant with hypertropic pyloric stenosis include metabolic alkalosis as a result of the vomiting that occurs in this disorder. Additional findings include decreased serum potassium and sodium levels, increased pH and bicarbonate level, and decreased chloride level. Options 1, 3, and 4 are incorrect

A child with developmental dysplasia of the hip (DDH) is placed in a Pavlik harness. The nurse is aware that with this type of device the child's legs are immobilized in what position? 1. Prone 2. Abduction 3. Adduction 4. Extension

2 The Pavlik harness consists of chest and shoulder straps and foot stirrups. It is a device used to correct hip dislocations in infants with developmental dysplasia of the hip, consisting of a set of straps that hold the hips in flexion and abduction. Therefore, options 1, 3, and 4 are incorrect.

A 6 month old child, with a nursing diagnosis of excess fluid volume, is being seen by the nurse. Which of the following signs/symptoms would the nurse expect to see? 1. Sunken fontanel 2. Marked weight gain 3. Soft eyeballs 4. High urine specific gravity

2. Marked weight gain is noted when a child is in a state of fluid volume excess

A 7-month-old child who has yet to have a cleft palate repaired is saying a few words. The child's lip is intact. Which of the following words would the nurse expect the child to have the most difficulty saying? 1. "Ma ma" 2. "Da da" 3. "Ba ba" 4. "Pa pa"

2. The child would have marked difficulty saying, "Da da." The child would be able to say the rest of the words.

The nurse collects a urine specimen preoperatively from a child with epispadias who is scheduled for surgical repair. When analyzing the results of the urinalysis, which should the nurse most likely expect to note? 1. Hematuria 2. Proteinuria 3. Bacteriuria 4. Glucosuria

3 Epispadias is a congenital defect involving abnormal placement of the urethral orifice of the penis. The urethral opening is located anywhere on the dorsum of the penis. This anatomical characteristic facilitates entry of bacteria into the urine. Options 1, 2, and 4 are not characteristically noted in this condition.

A home care nurse is teaching an adolescent with type 1 diabetes mellitus about insulin administration and rotation sites. Which statement, if made by the adolescent, would indicate effective teaching? 1. "I should use only my stomach and my thighs for injections." 2. "I need to use a different major site for each insulin injection." 3. "I need to use one major site for 2 to 3 weeks before changing major sites." 4. "I need to use the same major site for 1 month before rotating to another site."

3 To help decrease variations in absorption from day to day, the adolescent should use one major site for injections for 2 to 3 weeks before changing major sites. The injections are rotated to different locations within that major site. All other options are incorrect. The most efficient rotation plan involves giving about four to six injections in one area, each injection about 2.5 cm (1 inch) apart, or the diameter of the insulin vial from the previous injection, and then moving to another area.

The parents of a child with a cleft lip are concerned and ask the nurse when the lip will be repaired. With which statement should the nurse respond? 1. Cleft lip cannot be repaired. 2. Cleft-lip repair is usually performed by 6 months of age. 3. Cleft-lip repair is usually performed during the first weeks of life. 4. Cleft-lip repair is usually performed between 6 months and 2 years.

3 Cleft-lip repair is usually performed during the first few weeks of life. Early repair may improve bonding and makes feeding much easier. Revisions may be required at a later age. All other options are incorrect.

A nurse is admitting a baby to the newborn nursery who the nurse suspects may have congenital hypothyroidism. Which of the following findings has the nurse observed? SATA 1. clubfeet 2. cleft plate 3. protruding tongue 4. umbilical hernia 5. imperforate anus

3, 4 Protruding tongue & umbilical hernia are associated with CHT.

A child has been diagnosed with periorbital cellulitis. For which of the following signs/symptoms should the nurse assess? 1. Subconjunctival hemorrhages 2. Yellow-tinged sclerae 3. Bluish streaks in tissues surrounding the eye. 4. Absence of the red reflex during eye examinations.

3. Bluish streaks in tissues surrounding the eye are seen in children with periorbital cellulitis The other options are not related to periorbital cellulitis.

A nurse is preparing to care for a newborn infant following creation of a colostomy for the treatment of imperforate anus. In the immediate postoperative period, the nurse plans to inspect the stoma and expects to note which finding in the colostomy? 1. Bleeding 2. Gray in color 3. Dark blue in color 4. Red and edematous

4 A fresh colostomy stoma will be red and edematous, but this will decrease with time. The colostomy site will then be pink, without evidence of abnormal drainage, swelling, or skin breakdown. The colostomy should not be bleeding. A gray or dark blue stoma indicates insufficient circulation and should be reported to the health care provider (HCP) immediately.

A pediatric nurse educator provides a teaching session to the nursing staff regarding phenylketonuria. Which statement should the nurse educator include in the session? 1. "Treatment includes dietary restriction of tyramine." 2. "Phenylketonuria is an autosomal dominant disorder." 3. "Phenylketonuria primarily affects the gastrointestinal system." 4. "All 50 states require routine screening of all newborn infants for phenylketonuria."

4 All 50 states require routine screening in newborn infants. Phenylketonuria is an autosomal recessive disorder. Treatment includes dietary restriction of phenylalanine intake. Phenylketonuria is a genetic disorder that results in central nervous system damage from toxic levels of phenylalanine in the blood.

A child has a right femur fracture caused by a motor vehicle crash and is placed in skin traction temporarily until surgery can be performed. During assessment, the nurse notes that the dorsalis pedis pulse is absent on the right foot. Which action should the nurse take? 1. Administer an analgesic. 2. Release the skin traction. 3. Apply ice to the extremity. 4. Notify the health care provider (HCP).

4 An absent pulse to an extremity of the affected limb after a bone fracture could mean that the child is developing or experiencing compartment syndrome. This is an emergency situation, and the HCP should be notified immediately. Administering analgesics would not improve circulation. The skin traction should not be released without an HCP's prescription. Applying ice to an extremity with absent perfusion is incorrect. Ice may be prescribed when perfusion is adequate to decrease swelling.

A 14-month old child is in the hospital post-op from repair of congenital esophageal atresia (anastomosis of the ends of the esophagus). It is important for the nurse to encourage the surgeon to order a referral for the child to which of the following health-care practitioners? 1. Speech therapist 2. Stoma nurse 3. Otolaryngologist 4. Occupational therapist

4. The child will need occupational therapy. Speech Therapy and Otolaryngologist will not be required. The child will have had soma care for his or her gastrostomy tube since birth so a new referral should not be needed.

The clinic nurse is assessing a child for dehydration. The nurse determines that the child is moderately dehydrated if which finding is noted on assessment? 1. Oliguria 2. Flat fontanels 3. Pale skin color 4. Moist mucous membranes

1 In moderate dehydration, the fontanels would be slightly sunken, the mucous membranes would be dry, and the skin color would be dusky. Also, oliguria would be present.

The nurse is performing an assessment on a child admitted to the hospital with a probable diagnosis of nephrotic syndrome. Which assessment findings should the nurse expect to observe? Select all that apply. 1. Pallor 2. Edema 3. Anorexia 4. Proteinuria 5. Weight loss 6. Decreased serum lipids

1, 2, 3, 4 Nephrotic syndrome is a kidney disorder characterized by massive proteinuria, hypoalbuminemia, edema, elevated serum lipids, anorexia, and pallor. The child gains weight.

The nurse is educating the parents and their 10-year-old child regarding home care for the child's diagnosis of acute glomerular nephritis. Which of the following statements by the child indicate that the child understood the teaching? SATA 1. "I can't eat any potato chips or other salty foods." 2. "I can't go to school for a week because I am contagious." 3. "I won't be able to go back to soccer practice for a long time." 4. "When I get home, I will have to stay in bed, except when I need to go to the bathroom."

1, 3, and 4 1. This statement is true. Children with AGN are usually on salt restricted diets. 3. This statement is correct. Until the urinalyses are normal, children are restricted from participating in contact sports. 4. This statement is correct. The child will require frequent urinalyses and blood pressure assessments to monitor the progression of the disease. They are not contagious after 1 day on penicillin and they are rarely placed on strict bedrest.

The nurse assesses the following blood gas results on a child in the emergency department. Which of the following diagnoses is consistent with the data? Po2- 60 mm Hg Pco2- 50 mm Hg HCO3- 30 mEq/L Base excess -4 PH: 7.28 1. Metabollic acidosis 2. Metabolic alkalosis 3. Respiratory acidosis 4. Respiratory alkalosis

3. The child is in respiratory acidosis

A 6-year old child is being assessed by a nurse for possible signs of dehydration. Which of the following assessments should the nurse perform? 1. Patellar reflexes 2. Anterior fontanel tension 3. Skin turgor 4. Pupil reactivity to light

3. The child's skin turgor should be assessed Patellar reflexes are not assessed with hydration status. Anterior fontanelle tension should be assessed in infants and young toddlers, not 6 year olds. Pupil reactivity to light is not checked when assessing hydration status.

The nurse notes that a girl, 8 years old, is exhibiting signs of precocious puberty. If left untreated, the nurse is aware that the young girl is at high risk for which of the following complications? 1. Plagiocephaly 2. Short stature 3. Infertility 4. Endometriosis

2. Short stature is seen in children with precocious puberty

The nurse is preparing a plan of care for an infant who will be returning from the recovery room following the surgical repair of a cleft lip located on the right side of the lip. On return from the recovery room, the nurse should plan to place the infant in which position? 1. Prone and flat 2. Supine and flat 3. On the left side 4. On the right side

3 Following cleft lip repair, the infant should be positioned supine or on the side lateral to the repair to prevent the suture line from contacting the bed linens. Immediately after surgery, it is best to place the infant on the left side rather than supine to prevent aspiration if the infant vomits.

A nurse provides home care instructions to the mother of a child who had a cleft palate repair 4 days ago. Which statement by the mother indicates the need for further instruction? 1. "I will use a short nipple on the bottle." 2. "I need to buy some straws for drinking." 3. "I can give my child the pacifier in 2 weeks." 4. "I will give my child baby foods or baby food mixed with water."

2 The mother needs to be instructed that straws, pacifiers, spoons, and fingers must be kept away from the child's mouth for 7 to 10 days. Additionally, the mother should be advised to avoid taking an oral temperature. Options 1, 3, and 4 are accurate measures to implement after cleft palate repair.

A child is severely dehydrated from a diarrheal illness. The nurse assesses the child's lab results. Which of the following results would the nurse expect to find? 1. Hct 30% 2. Partial pressure of O2 60 mm Hg 3. Potassium 3.0 mEq/l 4. Plt count 100,000 cells/mm3

3. The nurse would expect to see a lab report that shows hypokalemia

A baby is admitted to the newborn nursery with a chordee penis. The nurse carefully assesses the baby for which of the following signs/symptoms? 1. Blood-tinged urine 2. Constant dripping of urine from the urethra 3. Absence of urinary output 4. Urine flowing from the under surface of the penis

4. Babies with a chordee penis are at high risk for hypospadias. Babies with a chordee penis are not at high risk for the other options.

A baby is admitted with a diagnosis of intussusception. Which of the following signs/symptoms would the nurse expect to see? 1. Projectile vomiting 2. Acute constipation 3. Explosive flatus 4. Currant jelly stools

4. Currant jelly stools often are seen in babies with intussusception. The other options are not characteristic of intussusception.

The school nurse is responsible for caring for a number of school children with type 1 diabetes. Before which of the following activities should the nurse make sure a child consumes a snack? The child who: 1. sculpts in art class. 2. plays in the band. 3. acts in the school play. 4. plays on the soccer team.

4. The child will need an extra snack before playing on the soccer team.

A child has just been diagnosed with celiac disease. Which of the following signs and symptoms would the nurse expect the parents to report in the child's history? SATA 1. Irritability 2. Failure to thrive 3. Abdominal pain 4. Excessive hunger 5. Recurring diarrhea

1,2,3 & 5 These are all conditions seen in a child with celiac disease.

An 11-month old child is seen in the primary health-care practitioner's office with a chief complain of loose stools. The child's temperature, heart rate, and respiratory rate are 98.9 F, 148 bp, and 46 rpm, respectively. Which of the following factors places this child at high risk for the nursing diagnosis: Deficient Fluid Volume? The child's SATA 1. age 2. HR 3. Temperature 4. Chief complaint 5. Respiratory rate

1,2,4 & 5 1. The child's age places the child at high risk for deficient fluid volume. 2. The child's HR places the child at high risk or deficient fluid volume. 4. The child's chief complaint places the child at high risk for deficient fluid volume. 5. The child's respiratory rate places the child at high risk for deficient fluid volume.

A 4-year-old child is seen at the primary health-care provider's office with vomiting and diarrhea for the past 24 hours. The primary health-care provider orders a number of interventions. If ordered, the nurse should question the administration of which of the following medications for the child? 1. Lomotil (diphenoxylate/atropine) 2. Zofran (ondansetron) 3. Reglan (metoclopramide) 4. Dramamine (dimenhydrinate)

1. Although the child does have diarrhea, Lomotil is not recommended to be given to children.

A 2-year-old child with acute diarrhea has been diagnosed with mild dehydration. Which rehydration methods would the nurse expect the health care provider to prescribe? 1. Increase intake of water with a diet high in carbohydrates. 2. Consume oral rehydration fluid, advancing to a regular diet. 3. Begin the BRAT diet (bananas, rice, apples, and toast or tea). 4. Begin fluid replacement immediately with intravenous fluids.

2 Mild dehydration is usually treated at home and consists of age-appropriate diet along with oral rehydration fluids. The BRAT diet does not provide the rehydration needed in a child who is dehydrated. Water does not provide electrolyte fluid replacement, a need during dehydration. Hospitalization is not required with mild dehydration.

A 12-year-old child has been diagnosed with athlete's foot. Which of the following information should the nurse include in the patient education regarding the disease? 1. The anaerobic bacteria that cause the infection must be treated with IV antibiotics. 2. Eradication of the infection can take may weeks of treatment. 3. Transmission of the mites is by direct, person-to-person contact. 4. The child must deprive the causative organism of oxygen by wearing shoes that are fully enclosed.

2. Eradication of the infection can take many weeks of treatment. The child will be prescribed either a topical or oral anti fungal medication. Although the infection is transmitted by direct, person-to-person contact, the causative organism is one of a number of fungi. The child should wear light-colored socks and shoes that provide good ventilation.

A 5-year-old child who was playing with matches is admitted to the pediatric emergency department. The child has blistered burns covering both anterior thighs. Which of the following responses is consistent with the child's presentation? The depth and extent of the burns are: 1. Depth 1st degree, extent 10% 2.. Depth 2nd degree, extent 7% 3. Depth 2nd degree, extent 18% 4. Depth 3rd degree, extent 3%

2. The child has a 2nd degree burn over approximately 7% of the body.

The nurse is educating a new mother of a child born with both a cleft lip and a cleft plate regarding formula feeding. Which of the following actions should the nurse include in her teaching session? SATA 1. Instruct the mother to add rice cereal to the formula. 2. Encourage the mother to cup feed her baby rather than to bottle feed. 3. Advice the mother to hold the baby in an upright position during feedings. 4. Advise the mother to feed the baby slowly to allow the baby time to swallow and to rest. 5. Notify the mother of the importance of giving the baby pain medicine before each feeding.

3. 4 The mother should be advised to hold the baby in an upright position during feedings. The mother should be advised to feed the baby slowly to allow the baby time to swallow and to rest. The rest of the options would not be advised.

An 8 year old child is seen in the pediatrician's office for primary nocturnal enuresis. Which of the following nursing diagnoses should the nurse include in the child's nursing care plan? 1. Overflow Urinary Incontinence 2. Risk for impaired skin integrity 3. Risk for Imbalanced Fluid Volume 4. Situational Low Self-Esteem

4. Situational Low Self-Esteem is an appropriate nursing diagnosis for the nurse to include in the care plan. The child is not experiencing overflow incontinence which results from an overly distended bladder. The child's skin integrity is intact. The child is not experiencing imbalanced fluid volume.

The parent of a 6-month-old calls the child's primary health-care provider and states "My child has had 5 loose stools since she woke up this morning. What should I do?" The other is exclusively breastfeeding her baby. Which of the following responses by the nurse is appropriate? 1. "Let's figure out what you may have eaten during the last day that could have caused the diarrhea." 2. "Continue to feed the baby breast milk and give oral rehydration therapy after each feeding." 3. "That's not that unusual for babies who are breast fed but do call again if the stools turn a green color." 4. "Bring the baby in for an appointment with the doctor so that we can weigh and check over the baby."

4. The baby does need to be weighed to determine whether the baby is dehydrated.

A 2-year old child has just been diagnosed with type 1 diabetes. The nurse is providing education to the parents regarding signs of hypoglycemia. Which of the following information should the nurse include in her teaching session? 1. Child's breath will smell like fruit. 2. Child will complain of excessive thirst. 3. Child will complain of sleepiness and will appear fatigued. 4. Child's behavior will resemble a burst of anger or a temper tantrum.

4. The child's behavior will resemble a burst of anger or a temper tantrum is the child is hypoglycemic. The other choices are indicative of hyperglycemia.

The clinic nurse is assessing jaundice in a child with hepatitis. Which anatomical area would provide the best data regarding the presence of jaundice? 1. The nail beds 2. The skin in the sacral area 3. The skin in the abdominal area 4. The membranes in the ear canal

1 Jaundice, if present, is best assessed in the sclera, nail beds, and mucous membranes. Generalized jaundice appears in the skin throughout the body. Option 3 is an inappropriate area to assess for the presence of jaundice.

Cerebral palsy (CP) is a term applied to a disorder that impairs movement and posture. The effects on perception, language, and intellect are determined by the type that is diagnosed. What are the potential warning signs of CP? Select all that apply. 1. The infant's arms or legs are stiff or rigid. 2. A high risk factor for CP is very low birth weight. 3. By 8 months of age, the infant can sit without support. 4. The infant has strong head control but a limp body posture. 5. The infant has feeding difficulties, such as poor sucking and swallowing. 6. If the infant is able to crawl, only one side is used to propel himself or herself.

1, 2, 5, & 6 Options 1, 2, 5, and 6 are potential warning signs of CP. By 8 months of age, if the infant cannot sit up without support, this would be considered a potential warning sign, because this developmental task should be completed by this time. The infant with a potential diagnosis of CP has poor head control by 3 months of age, when head control should be strong.

A nurse is assigned to care for a child who is scheduled for an appendectomy. Select the prescription(s) that the nurse anticipates will be prescribed. Select all that apply. 1. Initiate an IV line. 2. Maintain an NPO status. 3. Administer a Fleet enema. 4. Administer intravenous antibiotics. 5. Administer preoperative medications. 6. Place a heating pad on the abdomen to decrease pain.

1,2,4,5 Appendicitis is an inflammation of the appendix. When the appendix becomes inflamed or infected, perforation may occur within a matter of hours, leading to peritonitis, sepsis, septic shock, and potential death. IV fluids would be started, and the child would be NPO while awaiting surgery. Usually antibiotics are administered because of the risk of perforation. Prescribed preoperative medications most likely would be administered on call to the operating room. In the preoperative period, enemas or laxatives should not be administered. Additionally, heat is not applied to the abdomen. Any of these interventions can cause rupture of the appendix and resultant peritonitis.

A one-month-old baby has been admitted to the pediatric unit with a diagnosis of pyloric stenosis. Which of the following assessments is highest priority for the nurse to report to the baby's primary health-care provider? 1. Sunken fontanel 2. Undigested emesis 3. Apical HR of 156 bpm 4. Serum potassium of 3.6 mEq/dl

1. It is the highest priority for the nurse to report a sunken fontanel.

Four babies were delivered in the maternity unit during a 24-hour period. Which of the babies would the nurse most predict would exhibit cryptorchidism? 1. 34 weeks gestation, 2,200 grams, Apgar 9/9 2. 37 weeks gestation, 4,000 grams, Apgar 8/9 3. 39 weeks gestation, 3,500 grams, Apgar 7/8 4. 42 weeks gestation, 2,400 grams, Apgar 8/8

1. Most babies who are born with cryptorchidism are preterm By 36 weeks gestation, the testes should be descended.

A baby, 12 hours old, in the neonatal ICU, has been diagnosed with esophageal atresia with tracheoesophageal fistula. Which of the following assessments is highest priority for the nurse to make? 1. Quantity of nasogastric secretions 2. O2 saturation levels 3. Apical HR 4. Weight of wet diapers

2. Assessing O2 saturation is the highest priority

Four sick children with type 1 diabetes have been admitted to the hospital Which child is most at risk of developing hypoglycemia? The child with: 1. bacterial sepsis 2. intussusception 3. jaundice 4. chickenpox

1. The child with bacterial sepsis is most at high risk for developing hypoglycemia.

A 3-month-old child is being assessed in the emergency department. The child's lab results are: potassium 5.5 mEq/L and sodium 150 mEq/L. Which of the following is most likely the etiology of the child's results? 1. Baby is consuming concentrated formula that is not diluted with water. 2. Child has a cardiac defect. 3. Child has gastroenteritis. 4. Parent fed the baby large quantities of plain water on a hot summer day.

1. The most likely etiology is that the child is consuming concentrated formula that has not been diluted with water.

A child is admitted to the pediatric unit with a serum potassium level of 3.0 mEq/L. For which of the following complications should the nurse carefully monitor the child? 1. Dysrhythmias 2. Thirst 3. Seizures 4. Dry mucous membranes

1. The nurse should monitor the child for dysrhythmias.

The nurse, who is assessing the blood gas results of a young child in the emergency department, notes that the Pco2 is elevated ant ht the PH is low. The nurse will check to see if the child's body has attempted to compensate for the disturbance by doing which of the following? 1. Raising the serum bicarbonate levels 2. Raising the serum oxygen levels 3. Raising the serum carbonic acid levels 4. Raising the serum potassium levels

1. The nurse would assess to see if the serum bicarbonate levels are elevated

An infant born with an imperforate anus returns from surgery with a colostomy. The nurse assesses the stoma and notes that it is red and edematous. What is the best nursing action based on this finding? 1. Elevate the buttocks. 2. Document the findings. 3. Apply ice immediately. 4. Call the health care provide

2 A fresh colostomy stoma would be red and edematous, but this would decrease with time. The colostomy site then becomes pink without evidence of abnormal drainage, swelling, or skin breakdown. The nurse should document these findings because this is a normal expectation. Options 1, 3, and 4 are inappropriate and unnecessary interventions.

The nurse is providing discharge instructions to the mother of a child who had a cleft palate repair. Which statement should the nurse make to the mother? 1. "You should use a plastic spoon to feed the child." 2. "You need to use a short nipple on the child's bottle." 3. "You can allow the child to use a pacifier but only for 30 minutes at a time." 4. "You need to monitor the child's temperature for signs of infection using an oral thermometer."

2 A short nipple should be placed on the child's bottle, and the mother should be instructed to give the child baby food or baby food mixed with water. The mother should be instructed that straws, pacifiers, spoons, or fingers must be kept away from the child's mouth for 7 to 10 days after surgery. A pacifier should not be used for at least 2 weeks following the surgical repair. Additionally, the mother should be advised to avoid taking oral temperatures.

The nurse admits a child to the hospital with a diagnosis of pyloric stenosis. On assessment, which data would the nurse expect to obtain when asking the mother about the child's symptoms? 1. Watery diarrhea 2. Projectile vomiting 3. Increased urine output 4. Vomiting large amounts of bile

2 In pyloric stenosis, hypertrophy of the circular muscles of the pylorus causes narrowing of the pyloric canal between the stomach and the duodenum. Clinical manifestations of pyloric stenosis include projectile vomiting, irritability, hunger and crying, constipation, and signs of dehydration including a decrease in urine output.

The nurse is monitoring a child with burns during treatment for burn shock. The nurse understands that which assessment provides the most accurate guide to determine the adequacy of fluid resuscitation? 1. Skin turgor 2. Neurological assessment 3. Level of edema at burn site 4. Quality of peripheral pulses

2. Sensorium is an accurate guide to determine the adequacy of fluid resuscitation. The burn injury itself does not affect the sensorium, so the child should be alert and oriented. Any alteration in sensorium should be evaluated further. A neurological assessment would determine the level of sensorium in the child. Options 1, 3, and 4 would not provide an accurate assessment of the adequacy of fluid resuscitation.

The mother of a 10-year-old child telephones the child's primary health-care provider's office. The mother informs the nurse, " A spider bit my daughter a couple of days ago, and today it is looking really bad. The bite is oozing, and the skin around the bite is red and painful." Which of the following statements by the nurse is appropriate at this time? 1. "I bet the bite is infected with a dangerous bacteria. She must be seen immediately so that we can start her on antibiotics." 2. "I would like her to be seen today. Please cover the bite, and bring her in for an appointment." 3. "Spider bites are notorious for getting worse before they get better. It should clear up in a couple of days." 4. "It sounds like the bite has been inflamed. I want you to put warm compresses on it three times a day until it gets better."

2. This is the appropriate statement for the nurse to make. The lesion should be covered, and the child should be seen. The bite may be infected with MRSA. It is inappropriate, however, for the nurse to make frightening statements to the child's parent. The child should be seen. The HCP may order warm compresses to the area, but the child should be seen.

A 10-year-old child has cellulitis of the calf. Which of the following interventions should the nurse educate the parents to implement? 1. Have the child use crutches when ambulating. 2. Apply warm compresses to the inflamed area. 3. Measure the depth of edema each day the child is on antibiotics. 4. Locate and culture the item that punctured the child's skin

2. Warm compresses promote circulation to the area. The child may ambulate normally Some edema is noted, but it is not necessary to measure the depth of edema each day. The object that punctured the skin may not be known, and, even if it is, it is rarely cultured.

A nurse is caring for a hospitalized child who is receiving a continuous infusion of intravenous (IV) potassium for the treatment of dehydration. Which assessment finding requires the need to notify the health care provider? 1. Weight increase of 0.5 kg 2. Temperature of 100.8° F rectally 3. A decrease in urine output to 0.5 mL/kg/hr 4. Blood pressure (BP) unchanged from baseline

3 The priority assessment is to assess the status of urine output. Potassium should never be administered in the presence of oliguria or anuria. If urine output is less than 1 to 2 mL/kg/hr, potassium should not be administered. A slight elevation in temperature would be expected in a child with dehydration. A weight increase of 0.5 kg is relatively insignificant. A BP that is unchanged is a positive indicator unless the baseline was abnormal. However, there is no information in the question to support such data.

A nurse is providing instructions to the parents of a child with scoliosis regarding the use of a brace. Which statement by a parent indicates a need for further teaching? 1. "I cannot place powder under the brace." 2. "I need to place a soft shirt on my child under the brace." 3. "I need to be sure to apply lotion on the skin under the brace." 4. "I need to encourage my child to perform prescribed exercises."

3 The use of lotions or powders should be avoided with a brace because they can become sticky or cake under the brace, causing irritation. Options 1, 2, and 4 are appropriate interventions for the use of a brace on a child.

A 6-year-old child is admitted to the pediatric unit with a diagnosis of acute post streptococcal glomerular nephritis. Which of the following toys/activities would be most appropriate for the nurse to provide to the child? 1. Push and pull toy 2. Bean bags and target 3. Crayons and paper 4. Set of blocks

3. It would be most appropriate to provide the child with crayons and paper. The activity would not be too strenuous, and the child could press his or her feelings about being hospitalized in a drawing. Push and pull toys are appropriate for active toddlers. Bean bags would be appropriate for an active child who is angry at being confined to a bed. A let of blocks would be appropriate for an active child who could get down onto the floor and build a tower.

A 6 year old child with antistreptolysin antibodies and negative cultures is admitted to the pediatric unit with a diagnosis of acute post streptococcal glomerular nephritis. It would be most appropriate for the nurse to admit the child into which of the following rooms? 1. Isolation room on droplet isolation with no roommate. 2. Isolation room on droplet and contact isolation with a child with bronchiolitis. 3. Regular patient room with 8-year-old child in traction for a broken femur 4. Regular patient room with 6-year-old child with diabetes for insulin control

3. This would be the most appropriate room to place the child. Children in the early stages of AGN often remain inter beds because of marked fatigue. A child in traction would also be confined to his or her bed. Isolation is not needed with negative cultures. A child in the hospital for insulin control is likely up and about with not medically imposed or self-imposed activity restrictions. Although the children are the same age, their activity levels will be much different.

The nurse caring for a 14-year-old girl with diabetes insipidus understands which of the following about this disorder. 1. DI is treated on a short-term basis with hormone replacement therapy. 2. DI may cause anorexia if proper meal planning is not addressed. 3. DI is treated with vasopressin on a lifelong basis. 4. DI requires strike fluid limitation until it resolves.

3. Vasopressin is the treatment of choice. It is important for patients and parents to understand that DI is a lifelong disease.

The clinic nurse reviews the record of an infant and notes that the health care provider has documented a diagnosis of suspected Hirschsprung's disease. The nurse reviews the assessment findings documented in the record, knowing that which symptom most likely led the mother to seek health care for the infant? 1. Diarrhea 2. Projectile vomiting 3. Regurgitation of feedings 4. Foul-smelling ribbon-like stools

4 Hirschsprung's disease is a congenital anomaly also known as congenital aganglionosis or aganglionic megacolon. It occurs as the result of an absence of ganglion cells in the rectum and other areas of the affected intestine. Chronic constipation beginning in the first month of life and resulting in pellet-like or ribbon-like stools that are foul-smelling is a clinical manifestation of this disorder. Delayed passage or absence of meconium stool in the neonatal period is also a sign. Bowel obstruction especially in the neonatal period, abdominal pain and distention, and failure to thrive are also clinical manifestations. Options 1, 2, and 3 are not associated specifically with this disorder.

The nurse is preparing to care for a child with a diagnosis of intussusception. The nurse reviews the child's record and expects to note which symptom of this disorder documented? 1. Watery diarrhea 2. Ribbon-like stools 3. Profuse projectile vomiting 4. Bright red blood and mucus in the stools

4 Intussusception is a telescoping of one portion of the bowel into another. The condition results in an obstruction to the passage of intestinal contents. A child with intussusception typically has severe abdominal pain that is crampy and intermittent, causing the child to draw in the knees to the chest. Vomiting may be present, but is not projectile. Bright red blood and mucus are passed through the rectum and commonly are described as currant jelly-like stools. Watery diarrhea and ribbon-like stools are not manifestations of this disorder.

An 8-year-old child is admitted to the emergency department with burns over 30% of the body. Which of the following orders is highest priority for the nurse to perform? 1. Injection of tetanus booster 2. Debridement of the burns 3. Application of Silvadene ointment 4. Administration of IV fluids

4. Administration of IV fluids is the priority action.

A school age kid with phenylketonuria is eating lunch. The child has the following foods on the lunch plate. Which of the food choices should the nurse question the child for choosing? 1. Buttered baked potato 2. Salted stringed beans 3. Stewed Bing cherries 4. Fried chicken legs

4. Children with PKU may not consume animal protein.

A neonate, 3, 337 grams, has been diagnosed with congenital hypothyroidism. The neonatologist has ordered Synthoid (levothyroxine sodium) to be administered orally once each day beginning today. The recommended dosage of the medication is: infants and neonates birth to 3 months: 10 to 15/mcg/kg PO daily. Please calculate the safe maximum dosage of the medication for this neonate. If rounding is needed, please round to the nearest hundredths place. ___________________ mcg PO daily

50.66 mcg PO daily

A child is brought to the hospital emergency department for an injury to the lower right arm that occurred in a fall off a bicycle. On assessment the nurse notes that the skin at the site of the injury is intact. A fracture is suspected, and a radiograph is taken. The nurse can see on the radiograph viewer that the fracture of the bone is across the entire bone shaft with some possible displacement. What type of fracture should the nurse determine that this child has? 1. Simple fracture 2. Greenstick fracture 3. Compound fracture 4. Comminuted fracture

1 A simple fracture is a fracture of the bone across its entire shaft with some possible displacement but without breaking the skin. A greenstick fracture is an incomplete fracture that occurs through only a part of the cross section of the bone; one side of the bone is fractured, and the other side is bent. A compound fracture, also called an open or a complex fracture, is one in which the skin or mucous membrane has been broken, and the wound extends to the depth of the fractured bone. A comminuted fracture is a complete fracture across the shaft of the bone with splintering of the bone fragments.

The nurse is caring for a 1-year-old child after cleft palate repair. On completion of feeding, the nurse should plan for which appropriate nursing action? 1. Rinsing the mouth with water 2. Cleaning the mouth with diluted hydrogen peroxide 3. Using a soft lemon and glycerin swab to clean the mouth 4. Using cotton swabs saturated with half-strength povidone-iodine (Betadine) solution to clean the mouth

1 After cleft palate repair, the mouth is rinsed with water after feedings to clean the palate repair site. Rinsing food and residual sugars from the suture line reduces the risk of infection. Options 2, 3, and 4 are incorrect procedures. Hydrogen peroxide, lemon and glycerin, and povidone-iodine are not used because of their harmful effect on oral tissues and the suture site.

A clinic nurse is assessing the status of jaundice in a child with hepatitis. Which anatomical area will provide the best data regarding the presence of jaundice? 1. The nail beds 2. The skin in the sacral area 3. The skin in the abdominal area 4. The membranes in the ear canal

1 Jaundice, if present, is best assessed in the sclera, nail beds, and mucous membranes. Generalized jaundice will appear in the skin throughout the body. The membranes of the ear canal are not an appropriate area to assess for the presence of jaundice.

A child has recently been diagnosed with type 1 diabetes mellitus. Which of the following factors in his medical and family histories would the nurse expect to see? 1. Child's grandfather has been diabetic since childhood. 2. Child's body mass index is 30. 3. Child rarely engages in aerobic activities. 4. Child has recently gained 15 pounds.

1. The nurse would expect to see that the child has a direct relative who is or was a type 1 diabetic.

What should the parent of a child with diabetes insipidus (DI) be taught about administering desmopressin acetate nasal spray? SATA 1. The use of the flexible nasal tube. 2. Nasal congestion causes this route to be ineffective. 3. The medication should be administered every 48 hours. 4. The medication should be administered every 8 to 12 hours. 5. Overmedication results in signs of SIADH. 6. Nasal sprays do not always works as well as injections.

1. Administering desmopressin acetate per nasal spray is a means of providing the necessary medication in a steady state, if it is given using the flexible nasal tube every 8 to 12 hours. This decreases nasal irritation. 2. If the child becomes ill with rhinorrhea, the nasal spray all need to be administered via the buccal mucosa or rectum or the medication changed to tablets. 4. Administering desmopressin acetate per nasal spray is a means of providing the necessary medication in a steady state, if it is given using the flexible nasal tube every 8 to 12 hours. 5. Side effects of the desmopressin acetate are those of SIADH.

To determine whether a baby is allergic to foods, the nurse should educate parents to feed their babies employing which of the following procedures? 1. Babies' first foods should be either pureed apples or peaches. 2. The first time babies are fed solid foods, the babies should be at least 8 months of age. 3. Babies' first foods should be fed one at a time for 4 to 7 days each. 4. The first time babies are fed solid foods, the foods should be mixed with apple juice.

3. Babies first foods should be fed one at time for 4 to 7 days First foods should be nonallergenic cereals. Solids should be introduced at 6 months of age. The first time babies are fed, solid foods, the foods should be mixed with formula or breast milk.

The nurse in the delivery room suspects that a newly birthed baby may have an esophageal atresia with tracheoesophageal fistula because the baby is exhibiting which of the following signs and symptoms? 1. Palpable mass in the left lower quadrant 2. Blood-tinged vomitus 3. Pseudostrabismus 4. Copious quantities of oral mucus

4. Copious quantities of oral mucus is a classic sign of esophageal atresia with tracheoesophageal fistula. Palpable mass, Blood tinged vomitus & Pseudostrabsimus are unrelated to tracheoesophageal fistula.

The nurse has completed discharge teaching for the family of a 10-year-old diagnosed with DI. Which statement best demonstrates the family's correct understanding of DI? 1. "The disease was probably brought on by a bad diet and little exercise." 2. "Diabetes seems to run in my family, and that may be why my child has it." 3. "My child will need to check blood sugar several times a day." 4. "My child will have to use the bathroom more often than other children."

4. Despite the use of vasopressin to treat the symptoms of DI, breakthrough urination is likely.

A 2-year-old child is admitted to a hospital burn unit with partial- and full-thickness burns involving 35% of body surface area. After admission assessment and review of the health care provider's prescriptions, the priority nursing intervention should focus on which action? 1. Inserting a Foley catheter 2. Inserting a nasogastric tube 3. Sedating with morphine sulfate 4. Restricting intravenously administered fluids

1 A Foley catheter is inserted into the child's bladder so that urine output can be measured accurately each hour. A nasogastric tube may or may not be required, but this is not the priority intervention. Although pain medication may be required, the child should not be sedated. Intravenously administered fluids are not restricted and are administered at a rate sufficient to keep the child's urine output at 1 mL/kg of body mass per hour, thus reflecting adequate tissue perfusion.

A nursing student is caring for an infant with a respiratory infection and is monitoring for signs of dehydration. The nursing instructor asks the student to identify the most reliable method of determining fluid loss. Which statement by the student indicates an understanding of the method to determine fluid loss? 1. Monitor body weight. 2. Obtain a temperature. 3. Monitor intake and output. 4. Assess the mucous membranes.

1 Body weight is the most reliable method of measuring body fluid loss or gain. One kilogram of weight change represents 1 L of fluid loss or gain. Options 2, 3, and 4 are also appropriate measures to assess for dehydration, but the most reliable method is to monitor body weight.

A child who sustained a fractured ankle has a short leg cast applied, and the nurse provides home care instructions to the mother. The mother returns to the emergency department 16 hours later because the child is complaining of severe pain. The nurse notes that the child's toes are cool, pale, and puffy and that the child is agitated and crying loudly. The mother states, "I gave her the pain medication you sent with us just like you told us, and I have kept her foot up on two pillows since we left, except when she gets up to go to the bathroom. I don't understand why she hurts so much. Do something!" What is the most likely clinical situation that occurred? 1. Compartment syndrome 2. Inadequate pain medication 3. Skin breakdown around the cast edges 4. Noncompliance with home care instructions

1 Compartment syndrome occurs as a result of pressure buildup within a tissue compartment bound by anatomical structures such as fascia. With a fracture, this pressure increase may occur as a result of the intense inflammatory response or severe bleeding caused by the bone injury, even when diligent nursing care has been provided. Pain disproportionate to the injury despite analgesic administration is the classic sign of compartment syndrome. The nurse should constantly assess for this complication and should instruct the caregiver about the manifestations associated with this complication.

A nurse has provided dietary instructions to the mother of a child with celiac disease. The nurse determines that the mother understands the instructions when the mother states to include which food in the child's diet? 1. Corn 2. Wheat cereal 3. Rye crackers 4. Oatmeal biscuits

1 Dietary management is the mainstay of treatment in celiac disease. All wheat, rye, barley, and oats should be eliminated from the diet and replaced with corn and rice. Vitamin supplements, especially fat-soluble vitamins and folate, may be needed in the early period of treatment to correct deficiencies. These are likely to be lifelong restrictions; although small amounts of grains may be tolerated after the ulcerations have healed.

The nurse is developing a plan of care for a 6-year-old child diagnosed with acute glomerulonephritis. The nurse should include which priority intervention in the plan of care? 1. Encourage limited activity and provide safety measures. 2. Catheterize the child to monitor intake and output strictly. 3. Encourage the child to talk about feelings related to illness. 4. Encourage classmates to visit and to keep the child informed of school events.

1 Glomerulonephritis is a term that refers to a group of kidney disorders characterized by inflammatory injury in the glomerulus. In glomerulonephritis, activity is limited, and most children, because of fatigue, voluntarily restrict their activities during the active phase of the disease. Catheterization may cause infection. A 6-year-old should not be encouraged to talk about feelings and may not understand the illness. The child should be allowed to express feelings in other ways, such as play. Visitors should be limited to allow for adequate rest.

The nurse is planning care for a child with hemolytic-uremic syndrome who has been anuric and will be receiving peritoneal dialysis treatment. The nurse should plan to implement which measure? 1. Restrict fluids as prescribed. 2. Care for the arteriovenous fistula. 3. Encourage foods high in potassium. 4. Administer analgesics as prescribed.

1 Hemolytic-uremic syndrome is thought to be associated with bacterial toxins, chemicals, and viruses that result in acute kidney injury in children. Clinical manifestations of the disease include acquired hemolytic anemia, thrombocytopenia, renal injury, and central nervous system symptoms. A child with hemolytic-uremic syndrome undergoing peritoneal dialysis because of anuria would be on fluid restriction. Pain is not associated with hemolytic-uremic syndrome, and potassium would be restricted, not encouraged, if the child is anuric. Peritoneal dialysis does not require an arteriovenous fistula (only hemodialysis).

A child is diagnosed with Hirschsprung's disease. The nurse is teaching the parents about the cause of the disease. Which statement, if made by the parent, supports that teaching was successful? 1. "Special cells are not present in the rectum, which caused the disease." 2. "The protein part of wheat, barley, rye, and oats is not being digested fully." 3. "The disease occurs from increased bowel motility that leads to spasm and pain." 4. "The disease occurs because of inability to tolerate sugar found in dairy products."

1 Hirschsprung's disease also is known as congenital aganglionosis or megacolon. It results from the absence of ganglion cells in the rectum and, to various degrees, up into the colon. Option 2 describes celiac disease. Option 3 describes irritable bowel syndrome. Option 4 describes lactose intolerance.

The nurse is assisting a health care provider (HCP) examining an infant with developmental dysplasia of the hip perform an Ortolani maneuver. The nurse understands that this maneuver is performed for which purpose? 1. To assess for hip instability 2. To assess for movement of the hips 3. To push the femoral head out of the acetabulum 4. To ensure that hyperextension and full range of motion exist

1 In developmental dysplasia of the hip, the head of the femur is seated improperly in the acetabulum or hip socket of the pelvis. Ortolani's maneuver is a test to assess for hip instability. The examiner abducts the thigh and applies gentle pressure forward over the greater trochanter. A "clicking" sensation indicates a dislocated femoral head moving into the acetabulum. This maneuver does not assess for hip movement or ensure that hyperextension and full range of motion exist. Pushing the femoral head out of the acetabulum is not the purpose of Ortolani's maneuver.

The clinic nurse is obtaining data about a child with a diagnosis of lactose intolerance. Which data should the nurse expect to obtain on assessment? 1. Reports of frothy stools and diarrhea 2. Reports of foul-smelling ribbon stools 3. Reports of profuse, watery diarrhea and vomiting 4. Reports of diffuse abdominal pain unrelated to meals or activity1

1 Lactose intolerance causes frothy stools and diarrhea. Abdominal distention, crampy abdominal pain, and excessive flatus also may occur. Foul-smelling ribbon stool is a clinical manifestation of Hirschsprung's disease. Profuse, watery diarrhea and vomiting are clinical manifestations of celiac disease. Diffuse abdominal pain is a clinical manifestation of irritable bowel syndrome.

The clinic nurse is reviewing the health care provider's prescription for a child who has been diagnosed with scabies. Lindane has been prescribed for the child. The nurse questions the prescription if which is noted in the child's record? 1. The child is 18 months old. 2. The child is being bottle-fed. 3. A sibling is using lindane for the treatment of scabies. 4. The child has a history of frequent respiratory infections.

1 Lindane is a pediculicide product that may be prescribed to treat scabies. It is contraindicated for children younger than 2 years because they have more permeable skin, and high systemic absorption may occur, placing the children at risk for central nervous system toxicity and seizures. Lindane also is used with caution in children between the ages of 2 and 10 years. Siblings and other household members should be treated simultaneously. Options 2 and 4 are unrelated to the use of lindane. Lindane is not recommended for use by a breast-feeding woman because the medication is secreted into breast milk.

A nursing student is assisting a school nurse in performing scoliosis screening on the children in the school. The nurse assesses the student's preparation for conducting the screening. The nurse determines that the student demonstrates understanding of the disorder when the student states that scoliosis is characterized by which finding? 1. Abnormal lateral curvature of the spine 2. Abnormal anterior curvature of the lumbar spine 3. Excessive posterior curvature of the thoracic spine 4. Abnormal curvature of the spine caused by inflammation

1 Scoliosis is defined as an abnormal lateral curvature in any area of the spine. The region of the spine most commonly affected is the right thoracic area, where it results in rib prominence. Option 2 describes lordosis, which usually is exaggerated during pregnancy, in obesity, or in persons with large tumors. Option 3 describes kyphosis, which also is known as humpback. Scoliosis does not occur as a sequela of inflammation.

The nurse is teaching the parents of a child with growth hormone deficiency about preparing and administering synthetic growth hormone to the child. Which statement, if made by the parents, would indicate an understanding of the procedure? 1. "We will rotate injection sites." 2. "We will give the injection weekly on Monday." 3. "We will administer the injection every morning." 4. "We will store the mixed growth hormone in the medicine cabinet."

1 Synthetic growth hormone comes in a powdered form that must be diluted for administration. It is given as a subcutaneous injection six or seven times per week as prescribed at bedtime. Parents are taught that, once diluted, the hormone preparation is to be stored at 36° to 46° F (refrigerated). Injection sites should be rotated, which will direct you to the correct option.

A child with cerebral palsy (CP) is in a management program to achieve maximum potential for locomotion, self-care, and socialization in school. The nurse works with the child to meet these goals by performing which action? 1. Placing the child on a wheeled scooter board 2. Removing ankle-foot orthoses and braces once the child arrives at school 3. Keeping the child in a special education classroom with other children with similar disabilities 4. Placing the child in the supine position with a 30-degree elevation of the head of the bed to facilitate feeding

1 The correct option provides the child with maximum potential in locomotion, self-care, and socialization. While lying on the abdomen, the child can move around independently anywhere the child wants to go and can interact with others as desired. In option 2, orthoses need to be used all the time to aid locomotion. Option 3 does not provide for maximum socialization and normalization; rather, children with CP need to be mainstreamed as much as cognitive ability permits. Not all children with CP are intellectually challenged. Option 4 does not provide for normalization in self-care. Just as children without CP sit up and use assistive devices when eating, so should children with CP.

he nurse is planning discharge instructions for the mother of a child following orchiopexy, which was performed on an outpatient basis. Which is a priority in the plan of care? 1. Wound care 2. Pain control measures 3. Measurement of intake 4. Cold and heat applications

1 The most common complications associated with orchiopexy are bleeding and infection. Discharge instruction should include demonstration of proper wound cleansing and dressing and teaching parents to identify signs of infection, such as redness, warmth, swelling, or discharge. Testicles will be held in a position to prevent movement, and great care should be taken to prevent contamination of the suture line. Analgesics may be prescribed but are not the priority, considering the options presented. Measurement of intake is not necessary. Cold and heat applications is not a prescribed treatment measure.

The nurse is reinforcing instructions to the mother of a child who has a plaster cast applied to the left arm. Which statement by the mother indicates a need for further teaching? 1. "I will have to use a heat lamp to help the cast dry." 2. "I need to cover the cast with plastic during bathes or showers." 3. "I should call the health care provider if the cast feels warm or hot or has an unusual smell or odor." 4. "I will keep small toys and sharp objects away from the cast and be sure that my child does not put anything inside the cast."

1 The mother needs to be instructed not to use a heat lamp to help the cast dry because of the risk associated with a burn injury from the heat lamp. Options 2, 3, and 4 are appropriate instructions.

The mother of a child with an umbilical hernia calls the clinic and reports to the nurse that the child has been vomiting and is complaining of pain in the abdominal area. Which instruction to the mother is most appropriate? 1. Contact the health care provider. 2. Keep the child on clear liquids. 3. Apply an ice pack to the abdomen. 4. Administer acetaminophen (Tylenol).

1 Vomiting, pain, and irreducible mass at the umbilicus are signs of a strangulated hernia. The parents should be instructed to contact the health care provider (HCP) immediately if strangulation is suspected. Options 2, 3, and 4 are incorrect, can cause harm to the child, and delay emergency treatment measures that are required.

A nurse is assigned to care for a child following surgery to correct cryptorchidism. Which priority action should the nurse include in the plan of care following this type of surgery? 1. Prevent tension on the suture. 2. Force oral fluids, and monitor I&O. 3. Monitor urine for glucose and acetone. 4. Encourage coughing and deep breathing every hour.

1 When a child returns from surgery, the testicle is held in position by an internal suture that passes through the testes and scrotum and is attached to the thigh. It is important not to dislodge this suture, and it should be immobilized for 1 week. The most common complications are bleeding and infection. Depending on the type of anesthesia used, option 4 may be appropriate but it is not the priority for this type of surgery. Although it is important to maintain adequate hydration, it is inappropriate and unnecessary to force fluids. Testing urine for glucose and acetone also is not related to surgery.

A neighborhood nurse is attending a soccer game at a local middle school. One of the students falls off the bleachers and sustains an injury to the left arm. The nurse quickly attends to the child and suspects that the child's arm may be broken. Which nursing action would be the priority before transferring the child to the hospital emergency department? 1. Immobilize the arm. 2. Ask for the name of the child's pediatrician or family health care provider so that he or she can be contacted. 3. Have someone call the radiology department of the local hospital to let staff know that the child will be arriving. 4. Tell the child that the arm probably is fractured but not to worry because permanent damage to the arm will not occur.

1 When a fracture is suspected, it is imperative that the area be splinted and immobilized before the injured person is transferred or moved. The nurse should remain with the child and provide realistic reassurance. Although it may be necessary to contact the child's pediatrician, this is not the priority. It is not necessary to notify the radiology department because this would be the responsibility of the emergency department staff when the child arrives if it is determined that the child needs a radiograph. The child should not be told that permanent damage will not occur.

A 1-month-old infant is seen in a clinic and is diagnosed with developmental dysplasia of the hip. On assessment, the nurse understands that which finding should be noted in this condition? 1. Limited range of motion in the affected hip 2. An apparent lengthened femur on the affected side 3. Asymmetrical adduction of the affected hip when the infant is placed supine with the knees and hips flexed 4. Symmetry of the gluteal skinfolds when the infant is placed prone and the legs are extended against the examining table

1 n developmental dysplasia of the hip, the head of the femur is seated improperly in the acetabulum or hip socket of the pelvis. Asymmetrical and restricted abduction of the affected hip, when the child is placed supine with the knees and hips flexed, would be an assessment finding in developmental dysplasia of the hip in infants beyond the newborn period. Other findings include an apparent short femur on the affected side, asymmetry of the gluteal skinfolds, and limited range of motion in the affected extremity.

The nurse provides home care instructions to the parents of a child with celiac disease. The nurse should teach the parents to include which food item in the child's diet? 1. Rice 2. Oatmeal 3. Rye toast 4. Wheat bread

1 Celiac disease also is known as gluten enteropathy or celiac sprue and refers to an intolerance to gluten, the protein component of wheat, barley, rye, and oats. The important factor to remember is that all wheat, rye, barley, and oats should be eliminated from the diet and replaced with corn, rice, or millet. Vitamin supplements-especially the fat-soluble vitamins, iron, and folic acid-may be needed to correct deficiencies. Dietary restrictions are likely to be lifelong.

A nurse has been assigned to care for a neonate just delivered who has gastroschisis. Which concern should the nurse address in the client's plan of care? 1. Infection 2. Poor body image 3. Decreased urinary elimination 4. Cracking oral mucous membranes

1 Gastroschisis occurs when the bowel herniates through a defect in the abdominal wall to the right of the umbilical cord. There is no membrane covering the exposed bowel. Surgical repair will be done as soon as possible because of the risk of infection in the unprotected bowel. Therefore the highest risk immediately after delivery is infection. Because the client is a neonate, poor body image is not an immediate problem. Impaired urinary elimination is unlikely because the gastrointestinal tract is affected, not the genitourinary system. Gastroschisis involves the lower gastrointestinal system, so the oral mucous membranes are not affected.

The nurse is developing a plan of care for an infant after surgical intervention for imperforate anus. The nurse should include in the plan that which position is the most appropriate one for the infant in the postoperative period? 1. Prone position 2. Supine with no head elevation 3. Side-lying with the legs extended 4. Supine with the head elevated 45 degrees

1 The appropriate position following surgical intervention for an imperforate anus is a side-lying position with the legs flexed or a prone position to keep the hips elevated. These positions will reduce edema and pressure on the surgical site. Options 2, 3, and 4 will promote pressure at the surgical site.

A young girl is experiencing precocious puberty. Which of the following patient-care goals would be appropriate for the nurse to include in the child's plan of care? The young girl will: SATA 1. Wear age-appropriate attire 2. Shave axillary hair, as needed. 3. Not menstruate before age nine. 4. Have normal hormonal levels while receiving medication 5. State an understanding of the need for daily oral medications

1, 3, 4 The nurse would expect the child to wear age appropriate attire, not to menstruate before age 9 and to have normal hormonal levels while receiving medications. The nurse would not expect the child to shave her axillary hair. The mediations are administered intramuscularly usually once per month.

A child is brought to the emergency department, and diagnostic x-rays of the child reveal that a fracture is present. The mother states that the child was rollerblading and attempted to break a fall with an outstretched arm. A plaster of Paris cast is applied to the arm. Which instructions should the nurse provide the mother? Select all that apply. 1. The cast will mold to the body part. 2. The cast should be dry in about 6 hours. 3. Keep the cast elevated for the first day on pillows. 4. Make sure that the child can frequently wiggle the fingers. 5. The cast is water-resistant, so the child is able to take a bath or a shower. 6. The cast needs to be kept dry, because when wet it will begin to disintegrate.

1, 3, 4, 6 Options 1, 3, 4, and 6 are all important components of a teaching plan for a parent. Plaster of Paris is a heavier material than that used in a synthetic cast. It molds easily to the extremity and is less expensive than a synthetic cast. It takes about 24 hours to dry, but drying time could be longer, depending on the size of the cast. Plaster of Paris is not water resistant and when wet will begin to disintegrate. The cast should be elevated on a pillow for the first day to decrease swelling as the cast begins to mold to the arm. As the cast molds it is imperative that the child can wiggle the fingers because the extremity continues to swell. If the child can wiggle the fingers, adequate motion is present. Color and sensation of the fingers should also be assessed.

Which interventions should the nurse include when preparing a care plan for a child with hepatitis? Select all that apply. 1. Providing a low-fat, well-balanced diet. 2. Teaching the child effective hand-washing techniques. 3. Scheduling playtime in the playroom with other children. 4. Notifying the health care provider (HCP) if jaundice is present. 5. Instructing the parents to avoid administering medications unless prescribed. 6. Arranging for indefinite home schooling because the child will not be able to return to school.

1,2,5 Hepatitis is an acute or chronic inflammation of the liver that may be caused by a virus, a medication reaction, or another disease process. Because hepatitis can be viral, standard precautions should be instituted in the hospital. The child should be discouraged from sharing toys, so playtime in the playroom with other children is not part of the plan of care. The child will be allowed to return to school 1 week after the onset of jaundice, so indefinite home schooling would not need to be arranged. Jaundice is an expected finding with hepatitis and would not warrant notification of the HCP. Provision of a low-fat, well-balanced diet is recommended. Parents are cautioned about administering any medication to the child because normal doses of many medications may become dangerous owing to the liver's inability to detoxify and excrete them. Hand-washing is the most effective measure for control of hepatitis in any setting, and effective hand-washing can prevent the immunocompromised child from contracting an opportunistic type of infection.

The mother of a 3-year-old child arrives at a clinic and tells the nurse that the child has been scratching the skin continuously and has developed a rash. The nurse assesses the child and suspects the presence of scabies. The nurse bases this suspicion on which finding noted on assessment of the child's skin? 1. Fine grayish red lines 2. Purple-colored lesions 3. Thick, honey-colored crusts 4. Clusters of fluid-filled vesicles

1. Scabies is a parasitic skin disorder caused by an infestation of Sarcoptes scabiei (itch mite). Scabies appears as burrows or fine, grayish red, threadlike lines. They may be difficult to see if they are obscured by excoriation and inflammation. Purple-colored lesions may indicate various disorders, including systemic conditions. Thick, honey-colored crusts are characteristic of impetigo or secondary infection in eczema. Clusters of fluid-filled vesicles are seen in herpesvirus infection.

The nurse is educating the parents of a 2-month-old infant regarding the immunizations that the child will receive that day. The nurse should educate the parents that which of the following immunizations will protect the child form a serious GI infection? 1. Rotavirus (RV) 2.. Diphteria, tetanus, and acellular pertussis 3. Hib vaccine 4. Pneumococcal conjugate (PCV13)

1. Rotavirus is the only vaccine that would protect the child form a GI infection.

A 2-month old infant with a cleft lip is transferred to the pediatric floor immediately following surgical repair of the defect. Which of the following interventions should the nurse perform? 1. Assess placement of the elbow restraints. 2. Assess placement of the gastrostomy tube. 3. Monitor the child for signs of hypokalemia. 4. Monitor the child for passage of tarry stools.

1. The nurse should assess placement of the elbow restraints.. Gastrostomy tubes are not inserted in children with a diagnosis of cleft lip. The child is not at high risk for hypokalemia or passage of tarry stools.

A mother telephones the nurse at her child's primary health-care provider and states, "My child spilled my coffee on her arm. About one-half of the forearm is red, and there are 2 or 3 blisters that have developed. What should I do?" Which of the following is the best response for the nurse to give? 1. "Run cool water over the burned area and then call me back." 2. "Apply ice to the blisters for ten minutes on and ten minutes off." 3. "Proceed to the emergency department for a complete assessment." 4. "Cover the burned area with petroleum jelly and sterile bandages."

1. The skin should be cooled ASAP by running cool water over the burned area. Ice should not be applied to the skin. The ice can cause further damage to the skin. The child will likely be treated as an outpatient by the primary health-care provider. The nurse should, however, advise the parent to transport the child to the health-care providers's office after the burn has been called. Petroleum jelly should not be applied to the burned skin.

A nurse is caring for a child who fractured the ulna bone and had a cast applied 24 hours ago. The child tells the nurse that the arm feels like it is falling asleep. Which nursing action is most appropriate? 1. Encourage the child to keep the arm elevated. 2. Report the findings to the health care provider. 3. Document the findings, and reassess the arm 4 hours. 4. Tell the child that this is normal while the cast is drying.

2 A child's complaint of pins and needles or of the extremity falling asleep needs to be reported to the health care provider. These complaints indicate the possibility of circulatory impairment and paresthesia. Paresthesia is a serious concern because paralysis can result if the problem is not corrected. The five Ps of vascular impairment are pain, pallor, pulselessness, paresthesia, and paralysis. Prompt intervention is critical if neurovascular impairment is to be prevented.

An 18-month-old child is being discharged after surgical repair of hypospadias. Which postoperative nursing care measure should the nurse stress to the parents as they prepare to take their child home? 1. Leave the diapers off to allow the site to heal. 2. Avoid tub baths until the stent has been removed. 3. Encourage toilet training to ensure that flow of urine is normal. 4. Restrict fluid intake to reduce urinary output for the first few days.

2 After hypospadias repair, the parents are instructed to avoid giving the child a tub bath until the stent has been removed, to prevent infection. Diapers are placed on the child to prevent contamination of the surgical site. Toilet training should not be an issue during this stressful period. Fluids should be encouraged to maintain hydration.

A nursing student caring for a 6-month-old infant is asked to collect a sample for urinalysis from the infant. How should the student collect the specimen? 1. Catheterizing the infant using the smallest available Foley catheter 2. Attaching a urinary collection device to the infant's perineum for collection 3. Obtaining the specimen from the diaper by squeezing the diaper after the infant voids 4. Noting the time of the next expected voiding and then preparing a specimen cup for the urine

2 Although many methods have been used to collect urine from an infant, the most reliable method is the urine collection device. This device is a plastic bag that has an opening lined with adhesive so that it may be attached to the perineum. Urinary catheterization is not to be done unless specifically prescribed because of the risk of infection. Urine for certain tests, such as specific gravity, may be obtained from a diaper by collection of the urine with a syringe. It is not reasonable to try to identify the time of the next voiding to attempt to collect the specimen.

A child's fasting blood glucose levels range between 100 and 120 mg/dL daily. The before-dinner blood glucose levels are between 120 and 130 mg/dL, with no reported episodes of hypoglycemia. Mixed insulin is administered before breakfast and before dinner. The nurse should make which interpretation about these findings? 1. Exercise should be increased to reduce blood glucose levels. 2. Insulin doses are appropriate for food ingested and activity level. 3. Dietary needs are being met for adequate growth and development. 4. Dietary intake should be increased to avoid hypoglycemic reactions.

2 Blood glucose levels are a measure of the balance among diet, medication, and exercise. Options 1 and 4 imply that the data analyzed are abnormal. The question presents no data for determining growth and development status, such as height, weight, age, or behavior. Supporting normal growth and development is an important goal in managing diabetes in children, but that is not what is being evaluated here.

A child has just returned from surgery and has a hip spica cast. What is the nurse's priority action for this client? 1. Elevate the head of the bed. 2. Assess the circulatory status. 3. Abduct the hips using pillows. 4. Turn the child onto the right side.

2 During the first few hours after a cast is applied, the chief concern is swelling, which may cause the cast to act as a tourniquet and obstruct circulation. Therefore, circulatory assessment is a high priority. Elevating the head of the bed of a child in a hip spica cast would cause discomfort. Using pillows to abduct the hips is not necessary because a hip spica cast immobilizes the hip and knee. Turning the child side onto side at least every 2 hours is important because it allows the body cast to dry evenly and prevents complications related to immobility; however, it is not a higher priority than checking circulation.

The nurse is developing a plan of care for a 10-year-old girl with an exacerbation of eczema. Which problem should be addressed in the care for this child? 1. The client is at risk for infection related to viral lesions. 2. The client is at risk for infection related to scratching of pruritic lesions. 3. The client may have poor nutritional intake related to throat edema and mouth ulcers. 4. The client may have a negative body image related to the presence of thick, white crusty plaques over the elbows and knees.

2 Eczema is a superficial inflammatory process involving primarily the epidermis. The major goals of management are to relieve pruritus, lubricate the skin, reduce inflammation, and prevent and control secondary infection. Secondary infection can occur when areas affected by eczema are scratched as a result of the itching because open skin is a portal of entry for pathogens. The lesions are not viral, and they do not present as thick, white crusty plaques. They appear as red and scaly lesions that can weep, ooze, and crust. They commonly occur in the antecubital and popliteal areas. Throat edema and mouth ulcers are not characteristics of this disorder.

The nurse in the hospital is giving at-home feeding instructions to a family whose child is being discharged after being born with a cleft lip. Which statement by the mother would indicate that further teaching is indicated? 1. "I am so glad that I am able to breast-feed my baby." 2. "I must always feed my baby with a syringe and not use a nipple." 3. "I will burp my baby very frequently so that she does not swallow a lot of air." 4. "I will feed my baby while sitting in a chair and holding her more upright."

2 Infants with a cleft lip are fed using a special nipple. Therefore although all the interventions relate to feeding, option 2 should be clarified with the family because if they fed the baby using a syringe, the child's oral needs for sucking will not be met. Newborns should be burped frequently and fed in a somewhat upright position. These interventions are applicable to the child with a cleft lip as well. Breast-feeding is always an option and should be done unless the child is having difficulty. Most children with a small cleft lip can breast-fed.

A child admitted to the hospital with a diagnosis of gastroenteritis and dehydration weighs 17 pounds 2 ounces. The parents state that his preadmission weight was 18 pounds 4 ounces. Based on weight alone, what type of dehydration does the nurse expect? 1. Mild dehydration 2. Moderate dehydration 3. Severe dehydration 4. Acute dehydration

2 Mild dehydration is a weight loss of 3% to 5%; moderate dehydration is 6% to 10%; severe dehydration is greater than 10% weight loss. All types of dehydration are acute situations. The answer can be determined by calculating the percent of weight loss in dehydration. Because the math calculation determines more than a 5% weight loss but less than 10% weight loss, the correct answer is moderate dehydration. By calculating the percent of weight loss, the correct answer can be determined.

Which is a priority problem for a child with severe edema caused from nephrotic syndrome? 1. Risk for constipation 2. Risk for skin breakdown 3. Inability to regulate body temperature 4. Consumption of more calories or nutrients than the body requires

2 Nephrotic syndrome is a kidney disorder characterized by massive proteinuria, hypoalbuminemia (hypoproteinemia), and edema. A child with edema from nephrotic syndrome is at high risk for skin breakdown. Skin surfaces should be cleaned and separated with clothing to prevent irritation and resultant skin breakdown. The child will be anorexic, so "taking in more calories or nutrients than the body requires" is not a concern. A risk for constipation or inability to regulate body temperature is not a concern with nephrotic syndrome.

The nurse performing an admission assessment on a 2-year-old child who has been diagnosed with nephrotic syndrome notes that which most common characteristic is associated with this syndrome? 1. Hypertension 2. Generalized edema 3. Increased urinary output 4. Frank, bright red blood in the urine

2 Nephrotic syndrome is defined as massive proteinuria, hypoalbuminemia, hyperlipemia, and edema. Other manifestations include weight gain; periorbital and facial edema that is most prominent in the morning; leg, ankle, labial, or scrotal edema; decreased urine output and urine that is dark and frothy; abdominal swelling; and blood pressure that is normal or slightly decreased.

Russell's traction is prescribed for a child with a lower leg fracture. The mother of the child asks the nurse about the purpose of the traction. The nurse explains to the mother that which is the primary action of this type of traction? 1. Relieve the child's pain. 2. Reduce or realigns a fracture site. 3. Provide a form of restraint for the child. 4. Keep the child from moving around in bed.

2 Russell's traction uses skin traction to realign a fracture in the lower extremity and immobilize the hip and knee in a flexed position. It is important to keep the hip flexion at the prescribed angle to prevent fracture malalignment. The traction may also relieve pain by reducing muscle spasms, but this is not the primary reason for this traction. The child can still move in bed with some restriction as a result of the traction. Traction is never used to restrain a child.

The nurse is collecting data on a child brought to the health care clinic by the mother with a 1-week-old cat scratch. While assessing the scratch the nurse notes redness, heat, swelling, and red streaking surrounding the area. The child states that the scratch hurts. Cellulitis is diagnosed. When providing home care instructions, which statement by the mother indicates a need for further teaching? 1. "The child should rest in bed." 2. "I will apply cool moist soaks every 4 hours." 3. "I should take the child's temperature and watch for a fever." 4. "The affected extremity should be elevated and immobilized."

2 The child with cellulitis should rest in bed, and the affected extremity should be elevated and immobilized. Warm moist soaks applied every 4 hours increase circulation to the infected area, relieve pain, and promote healing. Frequent hand washing is essential to prevent the spread of infection. The child should be carefully monitored for signs of sepsis, increased fever, chills, and confusion, or for the spread of infection.

Parents bring their child to the emergency department and tell the nurse that the child has been complaining of colicky abdominal pain located in the lower right quadrant of the abdomen. The nurse suspects that the child has which disorder? 1. Peritonitis 2. Appendicitis 3. Intussusception 4. Hirschsprung's disease

2 The most common symptom of appendicitis is a colicky, periumbilical, or lower abdominal pain located in the right quadrant. Peritonitis is a complication that can follow organ perforation or intestinal obstruction. The classic signs and symptoms of intussusception are acute, colicky abdominal pain with currant jelly-like stools. Clinical manifestations of Hirschsprung's disease include constipation, abdominal distention, and ribbon-like, foul-smelling stools.

The nurse is caring for a child with a fracture who is placed in skeletal traction. The nurse should monitor for which sign of a serious complication associated with this type of traction? 1. Lack of appetite 2. Elevated temperature 3. Decrease in the urinary output 4. Increase in the blood pressure

2 The most serious complication associated with skeletal traction is osteomyelitis, an infection involving the bone. Organisms gain access to the bone systemically or through the opening created by the metal pins or wires used with the traction. Osteomyelitis may occur with any open fracture. Clinical manifestations include complaints of localized pain, swelling, warmth, tenderness, an unusual odor from the fracture site, and an elevated temperature. Options 1, 3, and 4 are not specifically associated with osteomyelitis.

A child sustains a fall at home and is brought to the hospital emergency department by the child's mother. After a radiographic examination, the child is determined to have a fractured arm, and a plaster cast is applied. The nurse provides instructions to the mother regarding neurocirculatory assessment and function. Which statement by the mother indicates a need for further instruction? 1. "I'll need to check her skin twice a day at the cast edges." 2. "If her hand gets real cool and pale, I can apply the heating pad to it." 3. "For the first couple of days, I should try to keep her hand higher than her heart most of the time, using pillows." 4. "If she seems way too fussy and her arm is painful even after I've given her the pain medication, it might be a problem, and I should call you for help to decide on what is happening."

2 The mother needs to understand that compartment syndrome is a complication of fracture and casting and can result in permanent limb damage as a result of pressure-related tissue necrosis. The extremity is elevated to prevent swelling, and the health care provider is notified immediately if any signs of neurovascular impairment develop. Cold fingers could indicate neurovascular impairment and should be reported. A heating pad is not applied to the cast or fingers. Skin edges are checked to monitor for irritation and skin breakdown.

The nurse is writing out discharge instructions for the parents of a child diagnosed with celiac disease. The nurse should focus primarily on which aspect of care? 1. Restricting activity 2. Following a gluten-free diet 3. Following a lactose-free diet 4. Giving medication to manage the condition

2 The primary nursing consideration in the care of a child with celiac disease is to instruct the child and parents regarding proper dietary management. Although medications may be prescribed for the client with celiac disease, treatment focuses primarily on maintaining a gluten-free diet. Options 1, 3, and 4 are not directly related to the care of a child with celiac disease.

The nurse is verifying that a mother understands how to care for her infant who has thrush. Which comment by the mother would indicate that further teaching is indicated? 1. "I will feed my baby before I apply the medication." 2. "I can put the medication in my son's bottle for him to drink." 3. "I need to thoroughly clean all bottles and nipples after every use." 4. "I will slowly put the medication in each cheek of my baby's mouth."

2 Thrush is a fungus caused by Candida albicans. The white patches resemble mild curds and are visible on the tongue, inner lips, oral mucosa, and gums. An antifungal topical medication may be prescribed and needs to be slowly administered in each cheek of the baby's mouth after eating to increase the time that the medication is in contact with the mucous membranes. It is not placed in the baby's bottle. This action does not provide the topical treatment needed; additionally, medication is not added to a child's bottle because the infant may not finish drinking everything from the bottle.

The nurse is teaching the parent of a preschool child how to administer the child's insulin injection. The child will be receiving 2 units of Humulin R insulin and 12 units of Humulin N insulin every morning. How should the nurse instruct the parents to prepare the insulin? 1. Draw the insulin into separate syringes. 2. Draw the Humulin R insulin first and then the Humulin N insulin into the same syringe. 3. Draw the Humulin N insulin first and then the Humulin R insulin into the same syringe. 4. Check blood glucose first, and if the result is between 80 and 120 mg/dL, withhold the insulin injection.

2 When mixing types of insulin, always withdraw the clear, rapid-acting insulin into the syringe first and then the long-acting insulin. This procedure avoids contaminating the short-acting insulin with the longer-acting insulin. Therefore the regular insulin would be drawn into the syringe first, followed by the NPH insulin. When a child's insulin dosage requires the injection of both short- and intermediate-acting insulin at the same time, it is preferable to mix the two and use a single injection. Blood glucose results between 80 and 120 mg/dL are considered to be euglycemic (normal), and the prescribed dose would be administered to maintain

A 4-year-old child sustains a fall at home and after an x-ray examination, the child is determined to have a fractured arm and a plaster cast is applied. The nurse provides instructions to the parents regarding care for the child's cast. Which statement by the parents indicates a need for further instruction? 1. "The cast may feel warm as the cast dries." 2. "I can use lotion or powder around the cast edges to relieve itching." 3. "A small amount of white shoe polish can touch up a soiled white cast." 4. "If the cast becomes wet, a blow drier set on the cool setting may be used to dry the cast."

2 eaching about cast care is essential to prevent complications from the cast. The parents need to be instructed not to use lotion or powders on the skin around the cast edges or inside the cast. Lotions or powders can become sticky or caked and cause skin irritation. Options 1, 3, and 4 are appropriate statements.

The clinic nurse is assessing a child suspected of having juvenile rheumatoid arthritis (JRA). Which assessment findings should the nurse expect to note in a child who has been diagnosed with JRA? Select all that apply. 1. Hematuria 2. Morning stiffness 3. Painful, stiff, and swollen joints 4. Limited range of motion of the joints 5. Stiffness that develops later in the day 6. History of late afternoon temperature, with temperature spiking up to 105° F

2, 3, 4, 6 Clinical manifestations associated with JRA include intermittent joint pain that lasts longer than 6 weeks and painful, stiff, and swollen joints that are warm to the touch, with limited range of motion. The child will complain of morning stiffness and may protect the affected joint or refuse to walk. Systemic symptoms include malaise, fatigue, lethargy, anorexia, weight loss, and growth problems. A history of a late afternoon fever with temperature spiking up to 105° F will also be part of the clinical manifestations.

The nurse caring for a child who sustained a burn injury plans care based on which pediatric considerations associated with this injury? Select all that apply. 1. Scarring is less severe in a child than in an adult. 2. A delay in growth may occur after a burn injury. 3. An immature immune system presents an increased risk of infection for infants and young children. 4. The lower proportion of body fluid to mass in a child increases the risk of cardiovascular problems. 5. Fluid resuscitation is unnecessary unless the burned area is more than 25% of the total body surface area. 6. Infants and young children are at increased risk for protein and calorie deficiency because they have smaller muscle mass and less body fat than adults.

2, 3, 6 Pediatric considerations in the care of a burn victim include the following: Scarring is more severe in a child than in an adult. A delay in growth may occur after a burn injury. An immature immune system presents an increased risk of infection for infants and young children. The higher proportion of body fluid to mass in a child increases the risk of cardiovascular problems. Burns involving more than 10% of total body surface area require some form of fluid resuscitation. Infants and young children are at increased risk for protein and calorie deficiencies because they have smaller muscle mass and less body fat than adults.

The nurse prepares a list of home care instructions for the parents of a child who has a plaster cast applied to the left forearm. Which instructions should be included on the list? Select all that apply. 1. Use the fingertips to lift the cast while it is drying. 2. Keep small toys and sharp objects away from the cast. 3. Use a padded ruler or another padded object to scratch the skin under the cast if it itches. 4. Place a heating pad on the lower end of the cast and over the fingers if the fingers feel cold. 5. Elevate the extremity on pillows for the first 24 to 48 hours after casting to prevent swelling. 6. Contact the health care provider (HCP) if the child complains of numbness or tingling in the extremity.

2, 5, 6 While the cast is drying, the palms of the hands are used to lift the cast. If the fingertips are used, indentations in the cast could occur and cause constant pressure on the underlying skin. Small toys and sharp objects are kept away from the cast, and no objects (including padded objects) are placed inside the cast because of the risk of altered skin integrity. The extremity is elevated to prevent swelling, and the HCP is notified immediately if any signs of neurovascular impairment develop. A heating pad is not applied to the cast or fingers. Cold fingers could indicate neurovascular impairment, and the HCP should be notified.

The lesion on a child's face has been diagnosed as impetigo. Which of the following information should the nurse educate the parents in relation to this problem? SATA 1. Child should refrain from bathing until the lesions are completely healed. 2. Crusts should be removed several times each day using contact precautions. 3. Child must be on antibiotics for at least twenty four hours before returning to school. 4. Meticulous hand washing must be maintained to prevent transmission to others in the family. 5. Safe dosage of benadryl (diphenhydramine) should be administered at bedtime until the lesions resolve.

2,3 & 4 2. The crust, caused by the oozing of vesicular fluid, should be removed when they form. Contact precautions should be maintained to prevent transmission. 3. After antibiotics have been taken for 24 hr, the child can return to school or camp. 4. Meticulous hand washing must be maintained to prevent transmission to others in the family.

A 2-year-old child with nephrotic syndrome is admitted to the pediatric unit. The following orders have been written in the child's medical record. Which of the actions is highest priority for the nurse to perform? 1. Place child on alternating pressure mattress. 2. Administer IV albumin 3. Weigh all wet diapers 4. Administer oral antibiotics

2. Administering IV albumin is the priority action. The other options are important but not priority actions.

A nurse is performing the newborn seen for phenylketonuria. Which of the following actions is the nurse performing? 1. Sending cord blood from delivery to the hospital laboratory. 2. Collecting blood from a heel stick on a two-day-old baby. 3. Placing a urine collection bag on the one-day-old baby. 4. Analyzing a baby's meconium stool under the microscope.

2. Blood sent by heel stick on a 2-day old baby would be sent for newborn screening.

A breast fed baby has thrush and a bright red diaper rash. The baby's mother is complaining of severe pain each time the baby feeds. The nurse suspects that which of the following organisms is likely responsible for these complaints? 1. Staphylococcus aureus 2. Candida albicans 3. Streptococcus pyogenes 4. Herpes simplex

2. C albicans causes thrush and bright red diaper rashes in neonates. Breastfeeding mothers whose nipple are infected with C. albicans complain of severe pain while feeding.

A nurse is caring for an infant who is very fussy and has a diagnosis of diabetes insipidus (DI). Which parameter should the nurse monitor while the infant is on fluid restrictions? 1. Oral intake. 2. Urine output. 3. Appearance of the mucous membranes. 4. Pulse and temperature.

2. It is crucial to monitor and record urine output. The infant with DI has hypo secretion of ADH, and fluid restriction has little effect on urine formation. This infant is at risk for dehydration and for fluid and electrolyte imbalances. Test taking hint: DI results from pituitary dysfunction. The posterior pituitary targets the renal tubules and acts on the distal and collecting tubules to make them permeable to water, thus increasing resorption and decreasing excretion of urine.

A 3-year-old child is admitted to the pediatric unit for surgery. The child has a tumor in his left kidney. The child is to undergo surgery the next day. Which of the following primary health-care practitioner prescriptions is most important for the nurse to follow? 1. Maintain the child NPO after midnight 2. Place a sign at the head of the bed stating, "Do not touch abdomen." 3. Send a urine specimen for a urinalysis. 4. Send a blood specimen for electrolyte analysis.

2. The nurse should first place a sign at the head of the bed stating "Do not touch abdomen." The other options are not the first priority

A 17-year-old young woman is being seen in the primary health-care provider's office for a chief complaint of acne. Which of the following diagnosis would be appropriate for the nurse to include to the client's plan of care? 1. Powerlessness 2. Risk for ineffective coping 3. Risk for Self mutilation 4. Self-neglect

2. The young woman is at risk for ineffective coping. Acne can be disfiguring, adversely affecting one's self esteem. This young woman has taken the initiatitive to be seen by a health-care practitioner for her acne. That action is not consistent with a nursing diagnosis of powerlessness. Although some patients do try to rupture the blemishes, that action is not consistent with a nursing diagnosis of self-mutilation The young woman has taken the initiative to be seen by a health care provider for her acne. That action is not consistent with a nursing diagnosis of self-neglect

A mother telephones her 8-month old baby's primary health care provider and informs the triage nurse, "My baby has a diaper rash. I have been putting baby powder on the rash, but it doesn't seem to be getting any better. What should I do?" Which of the following responses by the nurse is most appropriate? 1. "It is important that you stop using the powder. If the baby breathes it in, it will make the baby very sick." 2. "Exposing the rash to the air often helps. I would suggest leaving the baby's diaper off for ten minutes every few hours. That should help." 3. "I would suggest that you switch to cornstarch from the powder. The natural properties in the cornstarch are healing." 4. "I am making an appointment for the baby to be seen. It is very rare for babies to develop diaper rashes when they are at your baby's age."

2. This is an appropriate response. Although 1 is important to teach the mother it's better to use gentler language. It is recommended that cornstarch be used instead of powder but it doesn't have healing properties. Babies develop diaper rash more frequently after 6 months of age.

The parents of a Hispanic American child who has been diagnosed with Wilms' tumor ask the nurse about the origin of the tumor. Which of the following information should the nurse provide the parents? 1. "Nephroblastoma is a cancer that originated in another part of your child's body." 2. "The tumor often starts growing in the kidney while the baby is till in the uterus." 3. "Wilms' tumor is especially prevalent in the Hispanic population." 4. "The cancer is often seen in children who live in areas near nuclear reactors."

2. This statement is correct. Nephroblastomas arise from embryonic tissue and develop over time. Wilms' tumor is slightly more prevalent in the African American population.

The nurse advises the parents of a 1 1/2 year-old who is newly diagnosed with type 1 diabetes that he child's blood glucose level before dinner should be between 90 and 140 mg/dl. The mother states, "But that tis much higher than I read on an Internet Web site. " Which of the following responses by the nurse is appropriate? 1. "I am sorry, I was thinking of the level for after dinner. The correct before dinner level is 70 to 110 mg/dl." 2. "The level is higher that what you will usually see because young children's diets are not as predictable as the diets of older children and adults." 3. "The level before breakfast should be 70 to 110 mg/dl, but the before dinner level should be a higher level." 4. "You will find that your primary health care provider will change the level last each visit. The goal starts at a high level and drops as your child responds to the insulin."

2. This statement is correct. Toddlers often go through a stage when they are finicky eaters. They are, therefore, at high risk for becoming hypoglycemic. The higher preprandial blood glucose level helps to reduce the risk of developing low blood glucose levels. The other options are false.

An adolescent with type 1 diabetes mellitus is attending a dance in the school gym. The adolescent suddenly becomes flushed and complains of hunger and dizziness. The school nurse, who is present at the dance, takes the child to the nurse's office and performs a blood glucose level test that shows 60 mg/dL. Which is the initial nursing intervention? 1. Call the child's mother. 2. Assist the child with administering regular insulin. 3. Give the child ½ cup of a sugar-sweetened carbonated beverage. 4. Call an ambulance to take the child to the hospital emergency department.

3 A blood glucose level lower than 70 mg/dL indicates hypoglycemia. The child is attending an activity that is different from the normal routine at school. Insulin requirements change with unfamiliar situations. When signs of hypoglycemia occur, the child needs an immediate source of glucose. Regular insulin will lower the blood glucose level and is thus a harmful action. Although the child's mother will need to be notified of the occurrence, this is not the immediate action. There is no reason to take the child to the emergency department.

A 12-year-old girl is admitted to the hospital with suspected appendicitis. What nursing interventions should be implemented preoperatively? 1. Applying a heating pad for 5-minute intervals as prescribed 2. Administering acetaminophen (Tylenol) as needed for pain, as prescribed 3. Placing the adolescent in a fetal position, side-lying with legs drawn up to chest 4. Inserting a nasogastric tube and attaching it to low intermittent suction; measuring drainage as prescribed

3 A client with appendicitis is more comfortable when lying in what is traditionally known as the fetal position, with the legs drawn up toward the chest. This flexed positioning assists in decreasing the pain that comes with appendicitis by decreasing the pressure on the abdominal area. Option 1 describes an intervention that is contraindicated because heat can lead to a ruptured appendix. Option 2 is incorrect. Pain medications are not given to the client with acute appendicitis because they may mask the symptoms that accompany a ruptured appendix. Option 4 describes a nursing intervention that may be necessary postoperatively.

An adolescent with type 1 diabetes mellitus has been chosen for the school's cheerleading squad. The adolescent visits the school nurse to obtain information regarding adjustments needed in the treatment plan for diabetes. What should the school nurse instruct the student to do? 1. Eat half the amount of food normally eaten. 2. Take two times the amount of prescribed insulin on practice and game days. 3. Eat six graham crackers or drink a cup of orange juice prior to practice or game time. 4. Take the prescribed insulin 1 hour prior to practice or game time rather than in the morning.

3 An extra snack of 15 to 30 g of carbohydrate eaten before activities, such as cheerleader practice, will prevent hypoglycemia. Six graham crackers or a cup of orange juice will provide 15 to 30 g of carbohydrate. The adolescent should not be instructed to adjust the amount or time of insulin administration. Meal amounts should not be decreased.

A mother brings her child to the well-child clinic and expresses concern to the nurse because the child has been playing with another child diagnosed with hepatitis. The nurse performs an assessment on the child, knowing that which finding is unassociated with hepatitis? 1. Hepatomegaly 2. The presence of jaundice 3. The presence of left upper abdominal quadrant pain 4. The presence of dark-colored, frothy urine in the urine specimen

3 Assessment findings in a child with hepatitis include right upper quadrant tenderness and hepatomegaly. The stools will be pale and clay-colored, and urine will be dark and frothy. Jaundice may be present and will be best assessed in the sclerae, nail beds, and mucous membranes.

Parents bring their 2-week-old infant to a clinic for treatment after a diagnosis of clubfoot made at birth. Which statement by the parents indicates a need for further teaching regarding this disorder? 1. "Treatment needs to be started as soon as possible." 2. "I realize my infant will require follow-up care until fully grown." 3. "I need to bring my infant back to the clinic in 1 month for a new cast." 4. "I need to come to the clinic every week with my infant for the casting."

3 Clubfoot is a complex deformity of the ankle and foot that includes forefoot adduction, midfoot supination, hindfoot varus, and ankle equinus; the defect may be unilateral or bilateral. Treatment for clubfoot is started as soon as possible after birth. Serial manipulation and casting are performed at least weekly. If sufficient correction is not achieved in 3 to 6 months, surgery usually is indicated. Because clubfoot can recur, all children with clubfoot require long-term interval follow-up until they reach skeletal maturity to ensure an optimal outcome.

The nurse is caring for an infant with cryptorchidism. The nurse anticipates that the most likely diagnostic study to be prescribed would be the one that assesses which item? 1. Babinski reflex 2. DNA synthesis 3. Urinary function 4. Chromosomal analysis

3 Cryptorchidism (undescended testes) may occur as a result of hormone deficiency, intrinsic abnormality of a testis, or a structural problem. Diagnostic tests for this disorder are performed to assess urinary and kidney function because the kidneys and testes arise from the same germ tissue. Babinski reflex reflects neurological function. Assessing DNA synthesis and a chromosomal analysis are unrelated to this disorder.

The nurse has provided dietary instructions to the mother of a child with celiac disease. The nurse determines that further instruction is needed if the mother states that she will include which food item in the child's nutritional plan? 1. Corn 2. Chicken 3. Oatmeal 4. Vitamin supplements

3 Dietary management is the mainstay of treatment for the child with celiac disease. All wheat, rye, barley, and oats should be eliminated from the diet and replaced with corn and rice. Vitamin supplements, especially fat-soluble vitamins and folate, may be needed in the early period of treatment to correct deficiencies.

A 4-year-old child with acute glomerulonephritis is admitted to the hospital. The nurse identifies which client problem in the plan of care as the priority? 1. Infection related to hypertension 2. Injury related to loss of blood in urine 3. Excessive fluid volume related to decreased plasma filtration 4. Retarded growth and development related to a chronic disease

3 Glomerulonephritis is a term that refers to a group of kidney disorders characterized by inflammatory injury in the glomerulus. The child with acute glomerulonephritis will have an excessive accumulation of water and retention of sodium, leading to circulatory congestion and edema. Excessive fluid volume would be a focus for this disease process. No risk for infection is associated with this disease; it is a postinfectious process, usually from a pneumococcal, streptococcal, or viral infection. Hematuria is present, but the loss of blood is not enough to constitute a risk for injury. The disease is acute as opposed to chronic, and almost all children recover completely.

A nurse is collecting data on a child recently diagnosed with glomerulonephritis. Which question to the mother should elicit data associated with the cause of this disease? 1. "Has your child had any nausea or diarrhea?" 2. "Have you noticed any rashes on your child?" 3. "Did your child recently complain of a sore throat?" 4. "Did your child sustain any injuries to the kidney area?"

3 Group A beta-hemolytic streptococcal infection is a cause of glomerulonephritis. Often, the child becomes ill with streptococcal infection of the upper respiratory tract and then develops symptoms of acute poststreptococcal glomerulonephritis after an interval of 1 to 2 weeks. The questions to the mother in options 1, 2, and 4 are unrelated to a diagnosis of glomerulonephritis.

The nurse is caring for a 7-year-old child with glomerulonephritis and is preparing to discuss the plan of care with the parents. In anticipating this encounter, the nurse recognizes that which is a common reaction of parents to the diagnosis of glomerulonephritis? 1. Fear of the complicated treatment regimen 2. Anger at the child for requiring hospitalization 3. Guilt that they did not seek treatment more quickly 4. Depression that the child may not be able to play sports

3 Guilt is a common reaction of the parents of a child diagnosed with glomerulonephritis. Parents blame themselves for not responding more quickly to the child's initial symptoms, or they may believe they could have prevented the development of glomerular damage. Options 1, 2, and 4 may be associated with the parents' reaction to the diagnosis, but they are not common parental reactions.

A nursing student is caring for a hospitalized child who has hypotonic dehydration. The nursing instructor asks the student to describe this type of dehydration. Which statement by the student indicates that the student understands the physiology associated with this type of dehydration? 1. "It causes the serum sodium level to rise above 150 mEq/L." 2. "It occurs when water and electrolytes are lost in the same proportion." 3. "It occurs when the loss of electrolytes is greater than the loss of water." 4. "It occurs when the loss of water is greater than the loss of electrolytes."

3 Hypotonic dehydration occurs when the loss of electrolytes is greater than the loss of water; in this type of dehydration, the serum sodium level is less than 130 mEq/L. Isotonic dehydration occurs when water and electrolytes are lost in approximately the same proportion as they exist in the body. In this type of dehydration, the serum sodium levels remain normal (135 to 145 mEq/L). Options 1 and 4 describe hypertonic dehydration.

The school nurse has provided an instructional session about impetigo to parents of the children attending the school. Which statement, if made by a parent, indicates a need for further instruction? 1. "It is extremely contagious." 2. "It is most common in humid weather." 3. "Lesions most often are located on the arms and chest." 4. "It might show up in an area of broken skin, such as an insect bite."

3 Impetigo is a contagious bacterial infection of the skin caused by b-hemolytic streptococci or staphylococci, or both. Impetigo is most common during hot, humid summer months. Impetigo may begin in an area of broken skin, such as an insect bite or atopic dermatitis. Impetigo is extremely contagious. Lesions usually are located around the mouth and nose, but may be present on the hands and extremities.

A nurse is reviewing the health care provider's documentation in the record of a child admitted with a diagnosis of intussusception. The nurse expects to note that the health care provider has documented which manifestation? 1. Scleral jaundice 2. Projectile vomiting 3. Currant jelly stools 4. Pale-colored and hard stools

3 In the child with intussusception, bright red blood and mucus are passed through the rectum, resulting in what is commonly described as currant jelly stools. The child classically presents with severe abdominal pain that is crampy and intermittent, causing the child to draw the knees in to the chest. Vomiting may be present, but not projectile. Options 1 and 4 are not manifestations of this disorder.

The pediatric nurse educator provides a teaching session to the nursing staff regarding juvenile idiopathic arthritis (JIA). Which statement by a nursing staff member indicates a need for further education? 1. "A complication of JIA is iridocyclitis." 2. "JIA most often occurs before the age of 16." 3. "JIA is twice as likely to occur in boys than in girls." 4. "Clinical manifestations of JIA include morning stiffness and painful, stiff, swollen joints."

3 JIA is twice as likely to occur in girls than in boys. Options 1, 2, and 4 are accurate statements regarding this disorder.

The parents of a child with juvenile idiopathic arthritis call the clinic nurse because the child is experiencing a painful exacerbation of the disease. The parents ask the nurse if the child can perform range-of-motion exercises at this time. The nurse should make which response? 1. "Avoid all exercise during painful periods." 2. "Range-of-motion exercises must be performed every day." 3. "Have the child perform simple isometric exercises during this time." 4. "Administer additional pain medication before performing range-of-motion exercises."

3 Juvenile idiopathic arthritis is an autoimmune inflammatory disease affecting the joints and other tissues, such as articular cartilage. During painful episodes of juvenile idiopathic arthritis, hot or cold packs and splinting and positioning the affected joint in a neutral position help reduce the pain. Although resting the extremity is appropriate, beginning simple isometric or tensing exercises as soon as the child is able is important. These exercises do not involve joint movement.

A nurse reinforces instructions to the mother of a child diagnosed with pediculosis (head lice). Permethrin 1% (Nix) has been prescribed. Which statement by the mother regarding the use of the medication indicates a need for further teaching? 1. "I need to purchase the medication from the pharmacy." 2. "After rinsing out the medication, I need to avoid washing my child's hair for 24 hours." 3. "I need to shampoo my child's hair, apply the medication, and leave the medication on for 24 hours." 4. "I need to shampoo my child's hair, apply the medication, and leave it on for 10 minutes and then rinse it out."

3 Nix is an over-the-counter anti-lice product that kills both lice and eggs with one application and has residual activity for 10 days. It is applied to the hair after shampooing and left for 10 minutes before rinsing out. The hair should not be shampooed for 24 hours after the rinsing treatment.

The clinic nurse provides instructions to the parents of an infant with developmental dysplasia of the hip (DDH), regarding care of the Pavlik harness. Which instruction should the nurse include? 1. The harness should be worn 6 hours a day. 2. The infant should not be moved when out of the harness. 3. The harness should be removed to check the skin and for bathing. 4. The harness must be removed for diaper changes and feeding.

3 The Pavlik harness is used to treat developmental dysplasia of the hip. It provides support and stability to the hips. The Pavlik harness should be worn 16 to 23 hours a day and should be removed only to check the skin and for bathing. The infant may be moved when out of the harness, but the hips and buttocks should be supported carefully. The harness does not need to be removed for diaper changes or feedings.

The mother of a 6-year-old child who has type 1 diabetes mellitus calls a clinic nurse and tells the nurse that the child has been sick. The mother reports that she checked the child's urine and it was positive for ketones. The nurse should instruct the mother to take which action? 1. Hold the next dose of insulin. 2. Come to the clinic immediately. 3. Encourage the child to drink liquids. 4. Administer an additional dose of regular insulin.

3 When the child is sick, the mother should test for urinary ketones with each voiding. If ketones are present, liquids are essential to aid in clearing the ketones. The child should be encouraged to drink liquids. Bringing the child to the clinic immediately is unnecessary. Insulin doses should not be adjusted or changed.

An infant has just returned to the nursing unit after surgical repair of a cleft lip on the right side. The nurse should place the infant in which best position at this time? 1. Prone position 2. On the stomach 3. Left lateral position 4. Right lateral position

3 A cleft lip is a congenital anomaly that occurs as a result of failure of soft tissue or bony structure to fuse during embryonic development. After cleft lip repair, the nurse avoids positioning an infant on the side of the repair or in the prone position because these positions can cause rubbing of the surgical site on the mattress. The nurse positions the infant on the side lateral to the repair or on the back upright and positions the infant to prevent airway obstruction by secretions, blood, or the tongue. From the options provided, placing the infant on the left side immediately after surgery is best to prevent the risk of aspiration if the infant vomits.

The nurse is caring for a newborn with a suspected diagnosis of imperforate anus. The nurse monitors the infant, knowing that which is a clinical manifestation associated with this disorder? 1. Bile-stained fecal emesis 2. The passage of currant jelly-like stools 3. Failure to pass meconium stool in the first 24 hours after birth 4. Sausage-shaped mass palpated in the upper right abdominal quadrant

3 Imperforate anus is the incomplete development or absence of the anus in its normal position in the perineum. During the newborn assessment, this defect should be identified easily on sight. A rectal thermometer or tube may be necessary, however, to determine patency if meconium is not passed in the first 24 hours after birth. Other assessment findings include absence or stenosis of the anal rectal canal, presence of an anal membrane, and an external fistula to the perineum. Options 1, 2, and 4 are findings noted in intussusception.

A mother brings her 5-week-old infant to the health care clinic and tells the nurse that the child has been vomiting after meals. The mother reports that the vomiting is becoming more frequent and forceful. The nurse suspects pyloric stenosis and asks the mother which assessment question to elicit data specific to this condition? 1. "Are the stools ribbon-like and is the infant eating poorly?" 2. "Does the infant suddenly become pale, begin to cry, and draw the legs up to the chest?" 3. "Does the vomit contain sour undigested food without bile, and is the infant constipated?" 4. "Does the infant cry loudly and continuously during the evening hours but nurses or takes formula well?"

3 Option 3 presents classic symptoms of pyloric stenosis. Stools that are ribbon-like and a child who is eating poorly are signs of congenital megacolon (Hirschsprung's disease). An infant who suddenly becomes pale, cries out, and draws the legs up to chest is demonstrating physical signs of intussusception. Crying during the evening hours, appearing to be in pain, eating well, and gaining weight are clinical manifestations of colic.

The nurse should implement which interventions for a child older than 2 years with type 1 diabetes mellitus who has a blood glucose level of 60 mg/dL? Select all that apply. 1. Administer regular insulin. 2. Encourage the child to ambulate. 3. Give the child a teaspoon of honey. 4. Provide electrolyte replacement therapy intravenously. 5. Wait 30 minutes and confirm the blood glucose reading. 6. Prepare to administer glucagon subcutaneously if unconsciousness occurs.

3, 6 Hypoglycemia is defined as a blood glucose level less than 70 mg/dL. Hypoglycemia occurs as a result of too much insulin, not enough food, or excessive activity. If possible, the nurse should confirm hypoglycemia with a blood glucose reading. Glucose is administered orally immediately; rapid-releasing glucose is followed by a complex carbohydrate and protein, such as a slice of bread or a peanut butter cracker. An extra snack is given if the next meal is not planned for more than 30 minutes or if activity is planned. If the child becomes unconscious, cake frosting or glucose paste is squeezed onto the gums, and the blood glucose level is retested in 15 minutes; if the reading remains low, additional glucose is administered. If the child remains unconscious, administration of glucagon may be necessary, and the nurse should be prepared for this intervention. Encouraging the child to ambulate and administering regular insulin would result in a lowered blood glucose level. Providing electrolyte replacement therapy intravenously is an intervention to treat diabetic ketoacidosis. Waiting 30 minutes to confirm the blood glucose level delays necessary intervention.

The school nurse notifies the other of a 7-year-old girl that her child has head lice (pediculosis capitus). Which of the following information should the nurse advise the mother regarding the problem? 1. "I strongly suggest that you cut your child's hair short before using the lice medicine, and keep it short from now on." 2. "Your child will need to be kept at home until she has received the second treatment, one week after the first." 3. "After using the lice medicine, you will need to comb your child's hair with a fine-toothed comb." 4. "For up to three weeks after being treated with the lice medicine, your child may complain of itching."

3. After the treatment, the nits must be removed using a fine-toothed comb. It is not recommended for the child's hair to be cut. The child may return to school or camp once he or she has had one treatment. The child should not complain of itching once he or she has been treated.

A young girl is being discharged from the pediatric unit after a left nephrectomy for Stage 1 Wilms' tumor of the left kidney and the first round of chemotherapy. The nurse is providing the parents with discharge planning. Which of the following statements should the nurse include? 1. Child will need to restrict fluids for the rest of his or her life. 2. Child will require dialysis until a kidney for transplant is found. 3. Child will be able to live a normal life after the surgical site heals. 4. Child will have to take antirecession medications after surgery.

3. Child will be able to live a normal life after the surgical site heals. The child will not need to restrict fluids for the rest of his/her life. The child still has one kidney. There will be no need for dialysis. The child did not receive a transplant. The child will not need to take antirecession medications after surgery.

A 7 year old child has been prescribed desmopressin (DDAVP) 20 mcg intranasal (10 mcg in each nostril) for nocturnal enuresis. Which of the following information regarding the medication should the nurse include in the parent/child teaching session? 1. Child must consume at least five cups of fluid each day. 2. Medication should be stored in the freezer between administrations. 3. Severe headaches with blurred vision should be reported to the prescribing practitioner. 4. Spray should be administered into the nostrils while the child is lying supine with the head extended.

3. If the dosage is too high, the child may develop adverse signs, including severe high blood pressure with headaches and blurred vision. The child's fluid intake should be restricted, especially before bedtime. The medication should be kept in the refrigerator but should not be frozen. The child should be sitting upright, and the nasal spray bottle should be vertical during medication administration.

A school nurse is monitoring the eating patterns of a child with celiac disease. The nurse counsels the child to choose an alternate lunch when the child picks which of the following foods to put on the lunch tray? 1. Corn taco with refried beans 2. Rice noodles with beef and broccoli 3. Turkey meatloaf with baked potato 4. Roast pork with applesauce

3. The nurse should counsel a child with celiac disease who chooses meatloaf for lunch. The other options are compatible with a celiac diet.

A nurse is caring for an 18-month old child who is admitted to the pediatric unit with a diagnosis of diarrhea and a weight loss of 4%. The nurse notes that the child's serum sodium and potassium levels are: 140 mEq/l and 4.8mEq/l, respectively. Which of the following orders by the primary health-care provider would the nurse expect to receive? 1. Restriction of all dairy products 2. IV fluid with potassium added. 3. Feedings of oral rehydration therapy. 4. Bouillon soup for lunch and dinner.

3. The nurse would expect the child to be fed ORT. There is no indication in the question that the intake of dairy products would need to be restricted. The child's potassium level is normal IV potassium is not indicated. Bouillon soup contains high levels of sodium. A high salt intake is not indicated

A baby with a history of CF, is admitted to the emergency department. The baby is crying loudly and drawing his legs up toward his abdomen. A diagnosis o f intussusception is made. Which of the following orders would the nurse expect to receive at the time? 1. To administer a corticosteroid medication. 2. To prepare the baby for abdominal surgery. 3. To prepare the baby for an air enema. 4. To administer an antispasmodic medication

3. The nurse would expect to prepare the baby for an air enema. The other options are not indicated at this time.

A baby has been diagnosed with atopic dermatitis (eczema). Which of the following signs/symptoms would the nurse expect to see? 1. Macular rash on the baby's back and shoulders. 2. Vesicular rash over the baby's abdomen and perineum. 3. Weepy rash over both of the baby's forearms and cheeks. 4. Scaly rash on the baby's scalp and forehead.

3. The red, weepy rash usually is seen over both of the baby's forearms and cheeks.

The clinic nurse is educating the parents of a 10-year-old child with scabies regarding medication administration. Which of the following information should the nurse include in the teaching? 1. The child should have been bathed at lease 24 hours prior to the administration of the medication. 2. The oral medication must be administer on an empty stomach. 3. The topical medication must remain on the skin for a few hours. 4. The parent should re-administer the medication in one week if the child continues to complain of itching.

3. The topical medication must remain on the skin for 8 full hours. The child should bathe and throughly dry himself or herself shortly before the medication is administered. Topical medication is applied to the skin It is common for the itching to persist for 2 to 4 weeks after treatment.

A child is admitted to the pediatric unit. While the nurse was taking the nursing history, the chid regurgitated vomitus that looked like coffee grounds and smelled like feces. Which of the following communications would it be appropriate for the nurse to report to the primary health-care provider? "After assessing the vomitus, it appears that the child: 1. has an obstruction proximal to the stomach." 2. has a perforated duodenal ulcer." 3. is vomiting blood from the lower bowel." 4. is exhibiting signs of ruptured esophageal varices."

3. The vomitus does appear to included blood and feces from the lower bowel. Vomitus proximal to the stomach would appear as completely undigested food. Vomitus from a perforated duodenal ulcer would appear bile colored and mixed with blood. Blood-tinged vomitus from ruptured esophageal varies would appear bright red.

A primary health-care provider has ordered an IV of D5 1/2 NS for a child with a diagnosis of dehydration. The parent asks the nurse to explain why the child must receive the solution. Which of the following responses by the nurse is appropriate? 1. "The solution contains all of the substances that should be in your child's blood stream." 2. "The solution will replace the most important electrolytes that your child is missing." 3. "The fluid contains some sugar and some salt. Those, in addition to the fluid, will help to make your child better." 4. "The fluid is the same as the water that you drink. Your child needs the water in order to get better."

3. This is an appropriate response for the nurse to provide.

An Orthodox Jewish couple deliver a baby boy with hypospadias. The parents state, "We are so excited. We are planning the baby's bris (ritual circumcision) for next week. Which of the following responses by the nurse is appropriate? 1. "I know how happy you must be. I know that you will have a wonderful party." 2. "If you are comfortable sharing the information, what Hebrew name do you plan to give your baby next week?" 3. "I understand how important is is to have a bris, but the baby will not be able to be circumcised next week." 4. "Do you have a mohel to perform the bris? I know how hard it is to locate one who yu feel you can trust."

3. This is the most appropriate statement for the nurse to make. The baby will not be able to be circumcised at the bris.

A pediatric nurse is having a discussion with a father whose child has recently been diagnosed with spastic cerebral palsy. Which of the following statements by the nurse is appropriate? 1. "It must be very hard to know that your child's ability to move will decrease over time." 2. "I am sure that is is hard for you to know that your child has this disease, but at least the medicine will treat the underlying problem." 3. "The treatment plan for your child will focus on enabling him to have as normal movements as possible." 4. "The nerve stimulation of your child's legs will enable hi to walk on his own when he is older."

3. This statement is accurate

An emergency department nurse is performing an assessment on a child with a suspected diagnosis of intussusception. Which assessment question for the parents will elicit the most specific data related to this disorder? 1. "Does the child have any food allergies?" 2. "What do the bowel movements look like?" 3. "Has the child eaten any food in the last 24 hours?" 4. "Can you describe the type of pain that the child is experiencing?"

4 A report of severe colicky abdominal pain in a healthy, thriving child between 3 and 17 months of age is the classic presentation of intussusception. Typical behavior includes screaming and drawing the knees up to the chest. Options 1, 2, and 3 are important aspects of a health history but are not specific to the diagnosis of intussusception.

The nurse is reviewing the health care record of an infant suspected of having unilateral hip dysplasia. Which assessment finding should the nurse expect to note documented in the infant's record regarding this condition? 1. Full range of motion in the affected hip 2. An apparent short femur on the unaffected side 3. Asymmetrical adduction of the affected hip when placed supine, with the knees and hips flexed 4. Asymmetry of the gluteal skin folds when the infant is placed prone and the legs are extended against the examining table

4 Asymmetry of the gluteal skin folds when the infant is placed prone and the legs are extended against the examining table is noted in hip dysplasia. Asymmetrical abduction of the affected hip, when an infant is placed supine with the knees and hips flexed, would also be an assessment finding in hip dysplasia in infants beyond the newborn period. An apparent short femur on the affected side is noted, as well as limited range of motion.

A topical corticosteroid is prescribed by a health care provider for a child with atopic dermatitis (eczema). Which instruction should the nurse give the parent about applying the cream? 1. Apply the cream over the entire body. 2. Apply a thick layer of cream to affected areas only. 3. Avoid cleansing the area before application of the cream. 4. Apply a thin layer of cream and rub it into the area thoroughly.

4 Atopic dermatitis is a superficial inflammatory process involving primarily the epidermis. A topical corticosteroid may be prescribed and should be applied sparingly (thin layer) and rubbed into the area thoroughly. The affected area should be cleaned gently before application. A topical corticosteroid should not be applied over extensive areas. Systemic absorption is more likely to occur with extensive application.

The parents of a newborn have been told that their child was born with bladder exstrophy, and the parents ask the nurse about this condition. The nurse provides an accurate response based on what information? 1. A hereditary disorder that occurs in every other generation 2. Caused by the use of medications taken by the mother during pregnancy 3. A condition in which the urinary bladder is abnormally located in the pelvic cavity 4. An extrusion of the urinary bladder to the outside of the body through a defect in the lower abdominal wall

4 Bladder exstrophy is a congenital anomaly characterized by extrusion of the urinary bladder to the outside of the body through a defect in the lower abdominal wall. The cause is not known, and a higher incidence is seen in male newborns. Options 1, 2, and 3 are not characteristics of this disorder.

The nurse caring for an infant with a diagnosis of bladder exstrophy should implement which intervention in the care of the infant's exposed bladder? 1. Covering the bladder with a dry sterile dressing 2. Covering the bladder with a sterile gauze dressing 3. Applying sterile water soaks to the bladder mucosa 4. Covering the bladder with a sterile, nonadhering dressing

4 Care should be taken to protect the exposed bladder tissue from drying while allowing drainage of urine. This is best accomplished by covering the bladder with a sterile, nonadhering dressing. The use of gauze should be avoided because this type of dressing adheres to the mucosa and may damage the delicate tissue when removed. Sterile dressings and dressings soaked in solutions can also dry out and damage the mucosa when removed.

The parents of a child recently diagnosed with cerebral palsy ask the nurse about the disorder. The nurse bases the response on the understanding that cerebral palsy is which type of condition? 1. An infectious disease of the central nervous system 2. An inflammation of the brain as a result of a viral illness 3. A congenital condition that results in moderate to severe retardation 4. A chronic disability characterized by impaired muscle movement and posture

4 Cerebral palsy is a chronic disability characterized by impaired movement and posture resulting from an abnormality in the extrapyramidal or pyramidal motor system. Meningitis is an infectious process of the central nervous system. Encephalitis is an inflammation of the brain that occurs as a result of viral illness or central nervous system infection. Down syndrome is an example of a congenital condition that results in moderate to severe retardation.

The nurse is collecting data on an infant with a diagnosis of suspected Hirschsprung's disease. Which question to the mother will most specifically elicit information regarding this disorder? 1. "Does your infant have diarrhea?" 2. "Is your infant constantly vomiting?" 3. "Does your infant constantly spit up feedings?" 4. "Does your infant have foul-smelling, ribbon-like stools?"

4 Chronic constipation, beginning in the first month of life and resulting in pellet-like or ribbon stools that are foul-smelling, is a clinical manifestation of Hirschsprung's disease. Delayed passage or absence of meconium stool in the neonatal period is the primary sign. Bowel obstruction, especially in the neonatal period, abdominal pain and distention, and failure to thrive are also clinical manifestations. Options 1, 2, and 3 are not specific clinical manifestations of this disorder.

The nurse provided discharge instructions to the parents of a 2-year-old child who had an orchiopexy to correct cryptorchidism. Which statement by the parents indicate that further teaching is necessary? 1. "I'll check his temperature." 2. "I'll give him medication so he'll be comfortable." 3. "I'll check his voiding to be sure there's no problem." 4. "I'll let him decide when to return to his play activities."

4 Cryptorchidism is a condition in which one or both testes fail to descend through the inguinal canal into the scrotal sac. Surgical correction may be necessary. All vigorous activities should be restricted for 2 weeks after surgery to promote healing and prevent injury. This prevents dislodging of the suture, which is internal. Normally, 2-year-olds want to be active; allowing the child to decide when to return to his play activities may prevent healing and cause injury. The parents should be taught to monitor the temperature, provide analgesics as needed, and monitor the urine output.

A child with type 1 diabetes mellitus is brought to the emergency department by the mother, who states that the child has been complaining of abdominal pain and has been lethargic. Diabetic ketoacidosis is diagnosed. Anticipating the plan of care, the nurse prepares to administer which type of intravenous (IV) infusion? 1. Potassium infusion 2. NPH insulin infusion 3. 5% dextrose infusion 4. Normal saline infusion

4 Diabetic ketoacidosis is a complication of diabetes mellitus that develops when a severe insulin deficiency occurs. Hyperglycemia occurs with diabetic ketoacidosis. Rehydration is the initial step in resolving diabetic ketoacidosis. Normal saline is the initial IV rehydration fluid. NPH insulin is never administered by the IV route. Dextrose solutions are added to the treatment when the blood glucose level decreases to an acceptable level. Intravenously administered potassium may be required, depending on the potassium level, but would not be part of the initial treatment.

An adolescent client with type 1 diabetes mellitus is admitted to the emergency department for treatment of diabetic ketoacidosis. Which assessment findings should the nurse expect to note? 1. Sweating and tremors 2. Hunger and hypertension 3. Cold, clammy skin and irritability 4. Fruity breath odor and decreasing level of consciousness

4 Diabetic ketoacidosis is a complication of diabetes mellitus that develops when a severe insulin deficiency occurs. Hyperglycemia occurs with diabetic ketoacidosis. Signs of hyperglycemia include fruity breath odor and a decreasing level of consciousness. Hunger can be a sign of hypoglycemia or hyperglycemia, but hypertension is not a sign of diabetic ketoacidosis. Hypotension occurs because of a decrease in blood volume related to the dehydrated state that occurs during diabetic ketoacidosis. Cold clammy skin, irritability, sweating, and tremors all are signs of hypoglycemia.

Nursing care of the infant with eczema should focus on which action as a priority nursing intervention? 1. Keeping the infant content 2. Maintaining adequate nutrition 3. Applying antibiotic ointment to lesions 4. Preventing secondary infection of the lesions

4 Eczema is a superficial inflammatory process involving primarily the epidermis. The major goals of management are to relieve pruritus, lubricate the skin, reduce inflammation, and prevent or control secondary infections. Keeping the infant content and maintaining adequate nutrition are not priority care for an infant with eczema. Antibiotic ointment should be applied only for treatment of a secondary infection as prescribed by a health care provider.

The nurse is preparing to care for an infant who has esophageal atresia with tracheoesophageal fistula (TEF). Surgery is scheduled to be performed in 1 hour. Intravenous fluids have been initiated, and a nasogastric (NG) tube has been inserted by the health care provider. The nurse plans care, knowing that which intervention is of highest priority during this preoperative period? 1. Monitor the temperature. 2. Monitor the blood pressure. 3. Irrigate the NG tube every 5 to 10 minutes. 4. Aspirate the NG tube every 5 to 10 minutes.

4 Esophageal atresia with TEF represents a critical neonatal surgical emergency. While the infant is awaiting transfer to surgery, management centers on prevention of aspiration. The infant is kept supine or prone with the head of the bed elevated to decrease the chance that gastric secretions will enter the lungs. Intravenous fluids are essential. An NG tube must be in place and aspirated every 5 to 10 minutes to keep the proximal pouch clear of secretions. Monitoring the temperature and monitoring blood pressure are standard nursing interventions.

An infant is seen in the health care provider's office for complaints of frequent vomiting and spitting up after feedings. Findings indicate that the infant is not gaining weight and gastroesophageal reflux is suspected. Which would the nurse anticipate being prescribed initially in the care of this child? 1. Place in prone position after each feeding. 2. Administer omeprazole (Prilosec) before feeding. 3. Instruct parents to keep a log of feedings and any reflux present. 4. Change the formula to predigested formula and feed small, frequent feedings.

4 For infants with frequent vomiting and spitting up, the diagnosis of gastroesophageal reflux should be considered. The initial action is to alter the formula. After the formula is changed, the family will be instructed to keep a log of feedings and any reflux with the new formula. Medication is not started until after the formula is changed. A prone position increases the risk of reflux and thus aspiration.

A school-age child with type 1 diabetes mellitus has soccer practice three afternoons a week. The school nurse provides instructions regarding how to prevent hypoglycemia during practice. Which should the school nurse tell the child to do? 1. Eat twice the amount normally eaten at lunchtime. 2. Take half the amount of prescribed insulin on practice days. 3. Take the prescribed insulin at noontime rather than in the morning. 4. Eat a small box of raisins or drink a cup of orange juice before soccer practice.

4 Hypoglycemia is a blood glucose level less than 70 mg/dL and results from too much insulin, not enough food, or excessive activity. An extra snack of 15 to 30 g of carbohydrates eaten before activities such as soccer practice would prevent hypoglycemia. A small box of raisins or a cup of orange juice provides 15 to 30 g of carbohydrates. The child or parents should not be instructed to adjust the amount or time of insulin administration. Meal amounts should not be doubled.

The nurse is reviewing a treatment plan with the parents of a newborn with hypospadias. Which statement by the parents indicates their understanding of the plan? 1. "Caution should be used when straddling the infant on a hip." 2. "Vital signs should be taken daily to check for bladder infection." 3. "Catheterization will be necessary when the infant does not void." 4. "Circumcision has been delayed to save tissue for surgical repair."

4 Hypospadias is a congenital defect involving abnormal placement of the urethral orifice of the penis. In hypospadias, the urethral orifice is located below the glans penis along the ventral surface. The infant should not be circumcised because the dorsal foreskin tissue will be used for surgical repair of the hypospadias. Options 1, 2, and 3 are unrelated to this disorder.

The mother of an 18-month-old child tells the clinic nurse that the child has been having some mild diarrhea and describes the child's stools as "mushy." The mother tells the nurse that the child is tolerating fluids and solid foods. The most appropriate suggestion regarding the child's diet would be to give the child which items? 1. Jell-O, strained cabbage, and custard 2. Fluids only until the "mushy" stools stop 3. Rice and mashed potatoes diluted with skim milk 4. Applesauce, strained bananas, and strained carrots

4 If mild diarrhea occurs in a child younger than 2 years, a soft diet is advised as long as the child is tolerating solids. The ABCs (applesauce, strained bananas, and strained carrots), rice, potatoes, and other bland foods without dairy products are advised. Extra fluids may also be needed and may be given by adding 1 to 2 oz of additional water to each bottle of formula or juice.

A nurse is implementing a teaching plan for a 4-month-old child who has been diagnosed with developmental dysplasia of the hip (DDH). The child will be placed in the Pavlik harness. Which statement by the family indicates that they understand the care of their child while placed in the Pavlik harness? 1. "I know that the harness must be worn continuously." 2. "I will bring my child back to the orthopedic office in a month or two so the straps can be checked." 3. "I realize that I will also need to put two diapers on my child so that the harness will stay dry and does not get soiled." 4. "I will watch for any redness or skin irritation where the straps are applied and call the health care provider for red areas."

4 If stabilization of the hip is required, a cast is initially applied. This is kept in place for 3 to 6 months until the hip is stabilized. After this is completed, and if further treatment is required, a Pavlik harness is the treatment of choice next. A Pavlik harness is a removable abduction brace. This is a procedure that requires the brace be checked every 1 to 2 weeks for adjustment of the straps. The use of double diapering is not recommended for DDH because of the possibility of hip extension. Because there are straps applied to the child's skin, it is important to check the skin of the child frequently.

The nurse is counseling the young mother of a small child recently diagnosed with impetigo. The nurse should make which statement that provides the best information about impetigo? 1. "The main treatment while your daughter has impetigo will be to force fluids." 2. "Your daughter probably caught the impetigo because you don't wash her hands enough." 3. "There is no risk of passing impetigo to the other children once you begin the prescribed antibiotics." 4. "You will need to prevent any of the fluid from the blisters around your daughter's mouth from coming into contact with your other children, especially if they already have skin injuries."

4 Impetigo is a highly contagious bacterial infection of the skin. Fluids are important but are not a component of the main treatment for this infection. Additionally, fluids should never be forced. Although impetigo occurs in situations of poor hygiene, a judgmental statement as indicated in option 2 is inappropriate and nontherapeutic. The infection is communicable for 48 hours beyond initiation of antibiotic treatment.

A health care provider prescribes an intravenous (IV) solution of 5% dextrose and half-normal saline (0.45%) with 40 mEq of potassium chloride for a child with hypotonic dehydration. The nurse performs which priority assessment before administering this IV prescription? 1. Obtains a weight 2. Takes the temperature 3. Takes the blood pressure 4. Checks the amount of urine output

4 In hypotonic dehydration, electrolyte loss exceeds water loss. The priority assessment before administering potassium chloride intravenously would be to assess the status of the urine output. Potassium chloride should never be administered in the presence of oliguria or anuria. If the urine output is less than 1 to 2 mL/kg/hour, potassium chloride should not be administered. Although options 1, 2, and 3 are appropriate assessments for a child with dehydration, these assessments are not related specifically to the IV administration of potassium chloride.

A child is diagnosed with intussusception. On performing an assessment of the child, the nurse keeps in mind which finding as a characteristic of this disorder? 1. The presence of fecal incontinence 2. Incomplete development of the anus 3. The infrequent and difficult passage of dry stools 4. Invagination of a section of the intestine into the distal bowel

4 Intussusception is an invagination of a section of the intestine into the distal bowel. It is the most common cause of bowel obstruction in children aged 3 months to 6 years. The presence of fecal incontinence describes encopresis. Encopresis generally affects preschool and school-aged children. Incomplete development of the anus describes imperforate anus, and this disorder is diagnosed in the neonatal period. The infrequent and difficult passage of dry stools describes constipation. Constipation can affect any child at any time, although the incidence peaks at age 2 to 3 years.

A nurse is caring for a child who was brought to the clinic complaining of severe abdominal pain and is suspected of having acute appendicitis. The child is lying on the examining table, with the knees pulled up toward the chest. The nurse assists the health care provider with further assessment of the progression of the child's pain, knowing that the health care provider will palpate the abdomen in which location? 1. Midway between the liver and the gallbladder 2. Midway between the left iliac crest and the umbilicus 3. Midway between the left inguinal area and the acetabulum 4. Midway between the right anterior superior iliac crest and the umbilicus

4 McBurney's point is usually the location of greatest pain in the child with appendicitis. McBurney's point is midway between the right anterior superior iliac crest and the umbilicus. Options 1, 2, and 3 will not appropriately assess the progression of pain in the child with appendicitis.

The nurse recognizes that clinical manifestations of nephrotic syndrome include which findings? 1. Hematuria, bacteriuria, weight gain 2. Gross hematuria, albuminuria, fever 3. Hypertension, weight loss, proteinuria 4. Massive proteinuria, hypoalbuminemia, edema

4 Nephrotic syndrome is a kidney disorder. Clinical manifestations of nephrotic syndrome include edema, proteinuria, hypoalbuminemia, and hypercholesterolemia in the absence of hematuria and hypertension. No fever, bacteriuria, or weight loss would be noted with this syndrome.

The nurse is providing discharge instructions to the mother of a child with herpetic gingivostomatitis. Which response by the mother indicates the need for further teaching? 1. "I will offer my child soft, bland foods." 2. "I will encourage my child to drink fluids." 3. "I will give my child frozen ice pops to assist with fluid intake." 4. "I will not give my child anything to eat for 2 days to allow the lesions to heal and crust over."

4 Parents need to be reassured that a few days without solid food will not harm the child as long as fluid intake is adequate, but an NPO status is not appropriate. Parents should also be taught to contact the health care provider (HCP) if the child develops signs of dehydration. The child would not be kept NPO; in fact, dehydration is a concern with these children. Small feedings of bland soft foods should be offered to the child. Fluid intake is very important, and the child must be encouraged to drink. Frozen ice pops, non-citrus juices, and flat soft drinks are best.

The school nurse is conducting pediculosis capitis (head lice) assessments. Which finding indicates a child has a "positive" head check? 1. Maculopapular lesions behind the ears 2. Lesions in the scalp that extend to the hairline or neck 3. White flaky particles throughout the entire scalp region 4. White sacs attached to the hair shafts in the occipital area

4 Pediculosis capitis is an infestation of the hair and scalp with lice. The nits are visible and attached firmly to the hair shaft near the scalp. The occiput is an area in which nits can be seen. Maculopapular lesions behind the ears or lesions that extend to the hairline or neck are indicative of an infectious process, not pediculosis. White flaky particles are indicative of dandruff.

A 7-year-old child is seen in a clinic, and the primary health care provider documents a diagnosis of primary nocturnal enuresis. The nurse should provide which information to the parents? 1. Primary nocturnal enuresis does not respond to treatment. 2. Primary nocturnal enuresis is caused by a psychiatric problem. 3. Primary nocturnal enuresis requires surgical intervention to improve the problem. 4. Most children outgrow the bed-wetting problem without therapeutic intervention.

4 Primary nocturnal enuresis occurs in a child who has never been dry at night for extended periods. The condition is common in children, and most children eventually outgrow bed-wetting without therapeutic intervention. The child is unable to sense a full bladder and does not awaken to void. The child may have delayed maturation of the central nervous system. The condition is not caused by a psychiatric problem.

A child who has undergone spinal fusion for scoliosis complains of abdominal discomfort and begins to have episodes of vomiting. On further assessment, the nurse notes abdominal distention. On the basis of these findings, the nurse should take which action? 1. Administer an antiemetic. 2. Increase the intravenous fluids. 3. Place the child in a Sims's position. 4. Notify the health care provider (HCP).

4 Scoliosis is a three-dimensional spinal deformity that usually involves lateral curvature, spinal rotation resulting in rib asymmetry, and hypokyphosis of the thorax. A complication after surgical treatment of scoliosis is superior mesenteric artery syndrome. This disorder is caused by mechanical changes in the position of the child's abdominal contents, resulting from lengthening of the child's body. The disorder results in a syndrome of emesis and abdominal distention similar to that which occurs with intestinal obstruction or paralytic ileus. Postoperative vomiting in children with body casts or children who have undergone spinal fusion warrants attention because of the possibility of superior mesenteric artery syndrome. Options 1, 2, and 3 are incorrect.

A nurse is talking to the parents of a child newly diagnosed with diabetes mellitus. Which statement by the parents indicates an understanding of preventing and managing hypoglycemia? 1. "I will give 8 oz of diet cola at the first sign of weakness." 2. "I will administer glucagon immediately if shakiness is felt." 3. "I will report to the emergency department if the blood glucose level is 65 mg/dL." 4. "I will carry a glucose source when leaving home in case of a hypoglycemic reaction."

4 The child or parents should carry a source of glucose so it is readily available in the event of a hypoglycemic reaction. LifeSavers or hard candies will provide a source of glucose. A diet carbonated beverage does not meet this need. If the blood glucose level is 65 mg/dL, a source of glucose may be needed, but it is unnecessary to report to the emergency department. Glucagon is used for an unconscious client who is experiencing a hypoglycemic reaction and is unable to swallow.

The home care nurse is visiting a child newly diagnosed with diabetes mellitus. The nurse is instructing the child and parents regarding actions to take if hypoglycemic reactions occur. The nurse should tell the child to take which action? 1. Administer glucagon immediately if shakiness is felt. 2. Drink 8 ounces of diet cola at the first sign of weakness. 3. Report to a hospital emergency department if the blood glucose is 60 mg/dL. 4. Carry hard candies whenever leaving home in case a hypoglycemic reaction occurs.

4 The child should be instructed to carry a source of glucose for ready use in the event of a hypoglycemic reaction. Hard candies such as LifeSavers will provide a source of glucose. Glucagon is not administered if shakiness is felt but is used in an unconscious client or a person unable to swallow who is experiencing a hypoglycemic reaction. A diet beverage is sugar-free and will not be helpful. If the blood glucose level is 60 mg/dL, a source of glucose may be needed, but it is not necessary to report to the emergency department.

A 9-year-old child fractures the left tibia while using a skateboard. The nurse would be most concerned with the client's risk of future uneven leg length if the fracture was in which area of this long bone? 1. Epiphysis 2. Diaphysis 3. Metaphysis 4. Epiphyseal line

4 The epiphyseal line is the area that is responsible for longitudinal bone growth. A fracture affecting this area places the child at risk for uneven future growth if proper healing does not occur. The epiphyses are located at the proximal and distal ends of a bone and are the insertion sites for muscles. The diaphysis is the shaft or main longitudinal portion of a long bone. The metaphysis is an area of flaring of bone, located between the epiphysis and the diaphysis.

The mother of a newborn infant with hypospadias asks the nurse why circumcision cannot be performed. Which is the most appropriate response by the nurse? 1. "Circumcision will cause an infection." 2. "Circumcision is not performed in a newborn." 3. "Circumcision will cause difficulty with urination." 4. "Circumcision has been delayed to save tissue for surgical repair."

4 The infant should not be circumcised because the dorsal foreskin tissue will be used for surgical repair of the hypospadias. This defect will most likely be corrected during the first year of life to limit the psychological effects on the child. Options 1, 2, and 3 are inaccurate statements.

An adolescent is seen in the emergency department for a suspected sprain of the ankle. X-rays have been obtained, and a fracture has been ruled out. Which instruction should the nurse provide to the adolescent regarding home care for treatment of the sprain? 1. Elevate the extremity, and maintain strict bed rest for a period of 7 days. 2. Immobilize the extremity, and maintain the extremity in a dependent position. 3. Apply heat to the injured area every 4 hours for the first 48 hours, and then begin to apply ice. 4. Apply ice to the injured area for a period of 30 minutes every 4 to 6 hours for the first 24 to 48 hours.

4 The injured area should be wrapped immediately to support the joint and control the swelling. Ice is applied to reduce the swelling and should be applied for not longer than 30 minutes every 4 to 6 hours for the first 24 to 48 hours. The joint should be immobilized and elevated, but strict bed rest for a period of 7 days is not required. A dependent position will cause swelling in the affected area.

The nurse has been caring for an adolescent newly diagnosed with type 1 diabetes mellitus. The nurse provides instructions to the adolescent regarding the administration of insulin. The nurse should include which instruction? 1. Use only the stomach and thighs for injections. 2. Rotate each insulin injection site on a daily basis. 3. Use the same site for injections for 1 month before rotating to another site. 4. Use one major site for the morning injection and another site for the evening injection for 2 to 3 weeks before changing major sites.

4 To help decrease variations in absorption from day to day, the child should use one location within a major site for the morning injection, rotating to another site for the evening injection, and a third site for the bedtime injection if needed. This pattern should be continued for a period of 2 to 3 weeks before changing major sites. Options 1, 2, and 3 are incorrect instructions to the adolescent.

A child is placed in skeletal traction for treatment of a fractured femur. The nurse develops a plan of care and includes which intervention? 1. Ensure that all ropes are outside the pulleys. 2. Ensure that the weights are resting lightly on the floor. 3. Restrict diversional and play activities until the child is out of traction. 4. Check the health care provider's (HCP's) prescriptions for the amount of weight to be applied.

4 When a child is in traction, the nurse would check the HCP's prescription to verify the prescribed amount of traction weight. The nurse would maintain the correct amount of weight as prescribed, ensure that the weights hang freely, check the ropes for fraying and ensure that they are on the pulleys appropriately, monitor the neurovascular status of the involved extremity, and monitor for signs and symptoms of immobilization. The nurse would provide therapeutic and diversional play activities for the child.

A child has fluid volume deficit. The nurse performs an assessment and determines that the child is improving and the deficit is resolving if which finding is noted? 1. The child has no tears. 2. Urine specific gravity is 1.030. 3. Urine output is less than 1 mL/kg/hour. 4. Capillary refill is less than 2 seconds.

4 ndicators that fluid volume deficit is resolving would be capillary refill less than 2 seconds, specific gravity of 1.002 to 1.025, urine output of at least 1 mL/kg/hour, and adequate tear production. A capillary refill time less than 2 seconds is the only indicator that the child is improving. Urine output of less than 1 mL/kg/hour, a specific gravity of 1.030, and no tears would indicate that the deficit is not resolving.

The nurse provides feeding instructions to a parent of an infant diagnosed with gastroesophageal reflux disease. Which instruction should the nurse give to the parent to assist in reducing the episodes of emesis? 1. Provide less frequent, larger feedings. 2. Burp the infant less frequently during feedings. 3. Thin the feedings by adding water to the formula. 4. Thicken the feedings by adding rice cereal to the formula.

4 Gastroesophageal reflux is backflow of gastric contents into the esophagus as a result of relaxation or incompetence of the lower esophageal or cardiac sphincter. Small, more frequent feedings with frequent burping often are prescribed in the treatment of gastroesophageal reflux. Feedings thickened with rice cereal may reduce episodes of emesis. If thickened formula is used, cross-cutting of the nipple may be required.

After hydrostatic reduction for intussusception, the nurse should expect to observe which client response? 1. Abdominal distention 2. Currant jelly-like stools 3. Severe colicky-type pain with vomiting 4. Passage of barium or water-soluble contrast with stools

4 Intussusception is the telescoping of one portion of the bowel into another. Hydrostatic reduction may be necessary to resolve the condition. After hydrostatic reduction, the nurse observes for the passage of barium or water-soluble contrast material with stools. Options 1 and 2 are clinical indicators of intussusception. Option 3 is a sign of an unresolved gastrointestinal disorder.

A 1-year-old child is diagnosed with intussusception, and the mother of the child asks the nurse to describe the disorder. Which statement is correct about intussusception? 1. "It is an acute bowel obstruction." 2. "It is a condition that causes an acute inflammatory process in the bowel." 3. "It is a condition in which a distal segment of the bowel prolapses into a proximal segment of the bowel." 4. "It is a condition in which a proximal segment of the bowel prolapses into a distal segment of the bowel."

4 Intussusception occurs when a proximal segment of the bowel prolapses into a distal segment of the bowel. It is not an acute bowel obstruction, but it is a common cause in infants and young children. It is not an inflammatory process.

A home care nurse instructs the mother of a 5-year-old child with lactose intolerance about dietary measures for her child. The nurse should tell the mother that it is necessary to provide which dietary supplement in the child's diet? 1. Fats 2. Zinc 3. Protein 4. Calcium

4 Lactose intolerance is the inability to tolerate lactose, the sugar found in dairy products. Removing milk and other dairy products from the diet can provide adequate relief from symptoms. This, however, can result in a deficiency of calcium. Additional dietary changes may be required to provide adequate sources of calcium and, in the infant, protein, and calories.

The nurse is caring for an infant after repair of an inguinal hernia. Which of these assessment findings indicate that the surgical repair was effective? 1. A clean, dry incision 2. Abdominal distention 3. An adequate flow of urine 4. Absence of inguinal swelling with crying

4 With an inguinal hernia, inguinal swelling occurs when the infant cries or strains. Absence of this swelling would indicate resolution of this problem. A clean, dry incision refers to absence of wound infection after surgery. Abdominal distention indicates a continuing gastrointestinal problem. The flow of urine is not specific to an inguinal hernia.

Permethrin (Elimite) is prescribed for a child with a diagnosis of scabies. The nurse should give which instruction to the parents regarding the use of this treatment? 1. Apply the lotion to areas of the rash only. 2. Apply the lotion and leave it on for 6 hours. 3. Avoid putting clothes on the child over the lotion. 4. Apply the lotion to cool, dry skin at least 30 minutes after bathing.

4. Permethrin is massaged thoroughly and gently into all skin surfaces (not just the areas that have the rash) from the head to the soles of the feet. Care should be taken to avoid contact with the eyes. The lotion should not be applied until at least 30 minutes after bathing and should be applied only to cool, dry skin. The lotion should be kept on for 8 to 14 hours, and then the child should be given a bath. The child should be clothed during the 8 to 14 hours of treatment contact time.

The nurse has provided teaching to a mother whose 5-month old has been diagnosed with atopic dermatitis (eczema). Which of the following statements by the mother indicates that teaching was successful? 1. " I make sure that my baby is clothed warmly each day." 2. "My baby's favorite toy is a fuzzy teddy bear." 3. "Today, my baby is wearing a hand-knit wool sweater that my mother knit." 4. "Tomorrow, I plan to dress my baby in a cute cotton shirt and denim jeans."

4. It is recommended that babies with eczema be dressed in cotton clothing. The other options are incorrect. They often exacerbate the symptoms.


Related study sets

WorkBook Chapter 11 (DA1) Dental Assisting 1

View Set

project management chapter 3 quiz

View Set

Chapter 4: Writing Collaboratively

View Set

MO-MEGA Middle School Science- JK

View Set